0% found this document useful (0 votes)
42 views100 pages

MAE101

Uploaded by

vaanthanh2005
Copyright
© © All Rights Reserved
We take content rights seriously. If you suspect this is your content, claim it here.
Available Formats
Download as PDF, TXT or read online on Scribd
0% found this document useful (0 votes)
42 views100 pages

MAE101

Uploaded by

vaanthanh2005
Copyright
© © All Rights Reserved
We take content rights seriously. If you suspect this is your content, claim it here.
Available Formats
Download as PDF, TXT or read online on Scribd
You are on page 1/ 100

MAE101 - THEORY AND PROBLEMS

DINH PHUOC VINH

May 14, 2024

vinhdp2@fe.edu.vn
Contents
1 FUNCTIONS AND LIMITS 1
1.1 Functions . . . . . . . . . . . . . . . . . . . . . . . . . . . . . . . . . . . . . 1
1.2 The Limit of a Function . . . . . . . . . . . . . . . . . . . . . . . . . . . . . 1
1.3 Continuity . . . . . . . . . . . . . . . . . . . . . . . . . . . . . . . . . . . . . 2
1.4 Asymptotes . . . . . . . . . . . . . . . . . . . . . . . . . . . . . . . . . . . . 2
1.5 Solved Problems . . . . . . . . . . . . . . . . . . . . . . . . . . . . . . . . . 2
1.6 Supplementary Problems . . . . . . . . . . . . . . . . . . . . . . . . . . . . . 4
1.7 Multiple Choice Questions . . . . . . . . . . . . . . . . . . . . . . . . . . . . 5

2 DERIVATIVES 9
2.1 Derivatives and Rates of Change . . . . . . . . . . . . . . . . . . . . . . . . 9
2.2 Differentiation Formulas . . . . . . . . . . . . . . . . . . . . . . . . . . . . . 9
2.3 The Chain Rule . . . . . . . . . . . . . . . . . . . . . . . . . . . . . . . . . . 9
2.4 Implicit Differentiation . . . . . . . . . . . . . . . . . . . . . . . . . . . . . . 10
2.5 Linear Approximations and Differentials . . . . . . . . . . . . . . . . . . . . 10
2.6 Solved Problems . . . . . . . . . . . . . . . . . . . . . . . . . . . . . . . . . 11
2.7 Supplementary Problems . . . . . . . . . . . . . . . . . . . . . . . . . . . . . 15
2.8 Multiple Choice Questions . . . . . . . . . . . . . . . . . . . . . . . . . . . . 15

3 APPLICATIONS OF DIFFERENTIATION 19
3.1 Maximum and Minimum Values . . . . . . . . . . . . . . . . . . . . . . . . 19
3.2 Mean Value Theorem . . . . . . . . . . . . . . . . . . . . . . . . . . . . . . . 19
3.3 Optimization Problems . . . . . . . . . . . . . . . . . . . . . . . . . . . . . . 20
3.4 Newton’s Method . . . . . . . . . . . . . . . . . . . . . . . . . . . . . . . . . 20
3.5 Antiderivatives . . . . . . . . . . . . . . . . . . . . . . . . . . . . . . . . . . 21
3.6 Solved Problems . . . . . . . . . . . . . . . . . . . . . . . . . . . . . . . . . 21
3.7 Supplementary Problems . . . . . . . . . . . . . . . . . . . . . . . . . . . . . 26
3.8 Multiple Choice Questions . . . . . . . . . . . . . . . . . . . . . . . . . . . . 27

4 INTEGRALS 29
4.1 The Definite Integral . . . . . . . . . . . . . . . . . . . . . . . . . . . . . . . 29
4.2 The Fundamental Theorems of Calculus . . . . . . . . . . . . . . . . . . . . 30
4.3 The Substitution Rule . . . . . . . . . . . . . . . . . . . . . . . . . . . . . . 31
4.4 Solved Problems . . . . . . . . . . . . . . . . . . . . . . . . . . . . . . . . . 31
4.5 Supplementary Problems . . . . . . . . . . . . . . . . . . . . . . . . . . . . . 35
4.6 Multiple Choice Questions . . . . . . . . . . . . . . . . . . . . . . . . . . . . 36

5 TECHNIQUES OF INTEGRATION 41
5.1 Integration by Parts . . . . . . . . . . . . . . . . . . . . . . . . . . . . . . . 41
5.2 Approximate Integration . . . . . . . . . . . . . . . . . . . . . . . . . . . . . 41
5.3 Improper Integrals . . . . . . . . . . . . . . . . . . . . . . . . . . . . . . . . 42
5.4 Solved Problems . . . . . . . . . . . . . . . . . . . . . . . . . . . . . . . . . 43
5.5 Supplementary Problems . . . . . . . . . . . . . . . . . . . . . . . . . . . . . 47
5.6 Multiple Choice Questions . . . . . . . . . . . . . . . . . . . . . . . . . . . . 48

i
ii

6 SYSTEMS OF LINEAR EQUATIONS 51


6.1 Solutions and Elementary Operations . . . . . . . . . . . . . . . . . . . . . 51
6.2 Gaussian Elimination . . . . . . . . . . . . . . . . . . . . . . . . . . . . . . . 51
6.3 Homogeneous Equations . . . . . . . . . . . . . . . . . . . . . . . . . . . . . 52
6.4 Solved Problems . . . . . . . . . . . . . . . . . . . . . . . . . . . . . . . . . 53
6.5 Supplementary Problems . . . . . . . . . . . . . . . . . . . . . . . . . . . . . 56
6.6 Multiple Choice Questions . . . . . . . . . . . . . . . . . . . . . . . . . . . . 56

7 MATRIX ALGEBRA 59
7.1 Matrix Addition, Scalar Multiplication, and Matrix Multiplication . . . . . 59
7.2 Matrix Inverses . . . . . . . . . . . . . . . . . . . . . . . . . . . . . . . . . . 60
7.3 Linear Transformations . . . . . . . . . . . . . . . . . . . . . . . . . . . . . 61
7.4 Solved Problems . . . . . . . . . . . . . . . . . . . . . . . . . . . . . . . . . 62
7.5 Supplementary Problems . . . . . . . . . . . . . . . . . . . . . . . . . . . . . 66
7.6 Multiple Choice Questions . . . . . . . . . . . . . . . . . . . . . . . . . . . . 66

8 DETERMINANTS AND DIAGONALIZATION 71


8.1 Determinants . . . . . . . . . . . . . . . . . . . . . . . . . . . . . . . . . . . 71
8.2 Diagonalization and Eigenvalues . . . . . . . . . . . . . . . . . . . . . . . . 72
8.3 Solved Problems . . . . . . . . . . . . . . . . . . . . . . . . . . . . . . . . . 73
8.4 Supplementary Problems . . . . . . . . . . . . . . . . . . . . . . . . . . . . . 76
8.5 Multiple Choice Questions . . . . . . . . . . . . . . . . . . . . . . . . . . . . 77

9 THE VECTOR SPACE Rn 81


9.1 R3 and Cross Product . . . . . . . . . . . . . . . . . . . . . . . . . . . . . . 81
9.2 Subspaces and Spanning . . . . . . . . . . . . . . . . . . . . . . . . . . . . . 83
9.3 Independence and Dimension . . . . . . . . . . . . . . . . . . . . . . . . . . 83
9.4 Orthogonality . . . . . . . . . . . . . . . . . . . . . . . . . . . . . . . . . . . 84
9.5 Solved Problems . . . . . . . . . . . . . . . . . . . . . . . . . . . . . . . . . 84
9.6 Supplementary Problems . . . . . . . . . . . . . . . . . . . . . . . . . . . . . 89
9.7 Multiple Choice Questions . . . . . . . . . . . . . . . . . . . . . . . . . . . . 90

Bibliography 93
List of Tables

List of Figures
4.1 If f (x) ⩾ 0, RiemannRsum is the sum of areas of rectangles . . . . . . . . . 30
4.2 If f (x) ⩾ 0, integral ab f (x) dx is the area under the curve y = f (x) from
a to b. . . . . . . . . . . . . . . . . . . . . . . . . . . . . . . . . . . . . . . . 30

iii
iv
Chapter 1

FUNCTIONS AND LIMITS


“You do not really understand something unless you can explain it to your grandmother.”
— Albert Einstein

1.1 Functions

• A function f : A −→ B is a rule that assigns every element of A to exactly one


element of B. A is called the domain of f and B is called co-domain.

• The range of f is the set of all values of f (range = f (A) ⊂ B).

• The graph of a function f defined on D is the set {(x, f (x)) | x ∈ D}.

• A function f defined on domain D is called even if ∀x ∈ D, −x ∈ D and f (−x) =


f (x).

• A function f defined on domain D is called odd if ∀x ∈ D, −x ∈ D and f (−x) =


−f (x).

• Suppose c > 0 and the graph of f (x) is given, then the graph of

+ y = f (x) + c is obtained by shifting the graph of f c units upward


+ y = f (x + c) is obtained by shifting the graph of f c units to the left
+ y = −f (x) is obtained by reflecting the graph of about the x-axis.
+ y = f (−x) is obtained by reflecting the graph of about the y-axis.

• If u : A −→ B, f : B −→ C, the composition function f ◦ u is defined by

(f ◦ u)(x) := f (u(x)). (1.1)

1.2 The Limit of a Function

• If f is defined near the number a, then we write

lim f (x) = L
x→a

if we can make the value of f arbitrarily close to L by taking x to be sufficiently


close to a (on either side of ) but not equal to a.

• The limit of f as x approaches to a is unique (if it exists).

• limx→a f (x) = L ⇐⇒ limx→a+ f (x) = limx→a− f (x) = L.

1
2 FUNCTIONS AND LIMITS

• Some basic limits:


sin x 1
 x
lim = 1, lim 1+ = e.
x→0 x x→+∞ x
• Suppose near the number a we have f (x) ≤ g(x), then limx→a f (x) ≤ limx→a g(x)
• If f (x) ≤ g(x) ≤ h(x), and limx→a f (x) = limx→a h(x) = L, then limx→a g(x) = L.

1.3 Continuity
• The function f is called continuous at a if

lim f (x) = f (a) (1.2)


x→a

• f is called continuous on (a, b) if f is continuous at every number in (a, b).


• f is called discontinuous at a if it is not continuous at a.
• (Intermediate Value Theorem) Let f be a continuous function defined on a closed
interval [a, b]. If f (a) and f (b) are two real numbers such that f (a) < f (b) (or
f (a) > f (b)), then for any number y between f (a) and f (b), there exists at least
one c in the interval [a, b] such that f (c) = y.

1.4 Asymptotes
• (Vertical Asymptotes) A vertical asymptote of a function f (x) occurs at x = a if

lim f (x) = ±∞ or lim f (x) = ±∞


x→a− x→a+

• (Horizontal Asymptotes) A horizontal asymptote of a function f (x) occurs at y = L


if limx→±∞ f (x) = L.

1.5 Solved Problems



1°. Find the domain and the range of the function f (x) = 9 − x2 .
Solution. The function f is defined for 9 − x2 ≥ 0 ⇐⇒ −3 ≤ x ≤ 3. Hence, the
domain of f is [−3, 3].
Since 0 ≤ 9 − x2 ≤ 9 for x ∈ [−3, 3], the range of f is [0, 3].
2°. Show that the product of two odd functions defined on D is an even function.
Solution. Suppose f and g are two odd functions defined on D. Then, for all
x ∈ D, −x ∈ D and

f (−x) = −f (x), and g(−x) = −g(x).

Hence, (f · g)(−x) = f (x) · g(−x) = [−f (x)] · [−g(x)] = f (x)g(x) = (f · g)(x).


Therefore, f · g is an even function.
1.5 Solved Problems 3

3°. Given the graph of a function y = f (x). Describe how the graph of each the following
funcions are obtained from the graph of f .

(a) y = f (x + 3) + 2.
(b) y = f (|x| − 2).

Solution.

(a) To obtain the graph of y = f (x+3)+2, first shift the graph of f 2 units upward
and then 3 units to left.
(b) Notice that
f (x), x ≥ 0,
(
f (|x|) =
f (−x), x < 0.

Thererfore, to obtain the graph of y = f (|x| − 2), we follow several steps:


Step 1. Obtain the graph of y = f (|x|) by reflecting the part of the graph of f
when x < 0 about the y-axis and keep the remaining part of the graph of f .
Step 2. Obtain the graph of f (|x| − 2) by shifting the graph after Step 1 2 units
to the right.

4°. Find the limit or say that it does not exists.



1− x−2
lim
x→3 x−3

0
Solution. This limit is of the form when x = 3. Multiply the numerator and
√ 0
denominator by 1 + x − 2, we obtain
√ √
(1 − x − 2)(1 + x − 2) 12 − (x − 2) −(x − 3)
√ = √ = √
(x − 3)(1 + x − 2) (x − 3)(1 − x − 2) (x − 3)(1 + x − 2)
Now we can cancel the common factor of x − 3 and obtain

1− x−2 −(x − 3) −1
lim = lim √ = lim √
x→3 x−3 x→3 (x − 3)(1 + x − 2) x→3 1 + x − 2

Now, we can directly x = 3 into the expression and have



1− x−2 1
lim =− .
x→3 x−3 2

x − 4,

2
x ̸= 2,

5°. Consider the function f (x) = x−2
3,

x = 2.
Determine if the function f (x) is continuous at x = 2. If it is not continuous, identify
the type of discontinuity.
Solution.
To determine the continuity of f (x) at x = 2, we need to check three conditions
(1): f (2) is defined, (2): limx→2 f (x) exists, and (3): f (2) = limx→2 f (x).
4 FUNCTIONS AND LIMITS

Let’s check these conditions one by one


(1). f (2) = 3.
x2 − 4
(2). limx→2 f (x) = limx→2 = 4.
x−2
(3). Since f (2) is defined and limx→2 f (x) = 4 ̸= 3 = f (2), the function f (x) has a
removable discontinuity at x = 2.
Therefore, f (x) is not continuous at x = 2, but the discontinuity is removable.

6°. Consider the function f (x) = x3 − 3x2 + 3.


Show that there exists a root of f (x) in the interval [1, 2].
Solution. To apply the Intermediate Value Theorem, we need to check two condi-
tions:
1. f (x) is continuous on the closed interval [1, 2].
2. f (1) and f (2) have opposite signs.
1. Since f (x) = x3 − 3x2 + 3 is a polynomial function, it is continuous everywhere.
2. Direct compuatations yields f (1) = 1, f (2) = −1
Since f (1) = 1 and f (2) = −1, f (1) and f (2) have opposite signs.
By the Intermediate Value Theorem, since f (x) is continuous on [1, 2] and f (1) and
f (2) have opposite signs, there exists at least one c in the interval [1, 2] such that
f (c) = 0. This means that f (x) has at least one root in the interval [1, 2].
2x − 1
7°. Consider the function f (x) = .
x2 − 1
Determine the vertical and horizontal asymptotes, if any, of the function f (x).
Solution. To find the vertical asymptotes of f (x), we need to identify the values of
x for which the function becomes undefined.
The function f (x) is undefined when the denominator x2 − 1 is equal to zero. There-
fore, there are two vertical asymptotes at x = ±1.
To find the horizontal asymptote, we analyze the behavior of the function as x
approaches positive or negative infinity.
Let’s take the limit as x approaches positive infinity:
2x − 1 2
lim = lim =0
x→∞ x2 − 1 x→∞ x

Similarly,
2x − 1 2
lim = lim =0
x→−∞ x − 1
2 x→−∞ x

Therefore, f (x) has one horizontal asymptote y = 0.

1.6 Supplementary Problems

1°. Find the domain and the range of each of these function
1.7 Multiple Choice Questions 5


(a) f (x) = 3 − cos x.
(b) f (x) = ln(−x2 + 2x).

2°. Is there any function that is both odd and even? Why or why not?

3°. Given the graph of a function y = f (x). Describe how the graph of each the following
funcions are obtained from the graph of f .

(a) y = f (−x + 3).


(b) y = |f (x + 5)|.

4°. Find the limit or say that it does not exists.

x2 − 9
lim
x→3 |x − 3|

x2 −9
x ̸= 3,
(
5°. Consider the function f (x) = x−3 ,
ax + 2, x = 3.
Find all values of the number a such that the function f (x) is continuous on R.

6°. Consider the function f (x) = x3 − 4x2 + 2.


Show that there exists a root of f (x) in the interval [0, 1].

7°. Determine the vertical and horizontal asymptotes, if any, of the function f (x) =
2x
√ .
x2 + 1

1.7 Multiple Choice Questions



1. What is the domain of the function f (x) = x2 − 4?

(A) (−∞, −2) ∪ (2, ∞)


(B) (−∞, −2] ∪ [2, ∞)
(C) (−∞, 2) ∪ (2, ∞)
(D) (−∞, −2) ∪ (−2, ∞)
3 + cos x
2. Which of the following is the range of the function g(x) = ?
4 − sin2 x
2
 
(A) ,1
3
1
 
(B) ,1
2
1 4
 
(C) ,
2 3
2 4
 
(D) ,
3 3
3. Which of the following functions is odd?
6 FUNCTIONS AND LIMITS

(A) f (x) = x2
(B) g(x) = x sin(x)
(C) h(x) = ex
(D) k(x) = x3 cos(x)

4. Which of the following functions is even?

(A) f (x) = x2 − 4
(B) g(x) = cos(x) sin(x)
(C) h(x) = ex
(D) k(x) = x3 − 2x

5. Calculate the limit


x+2
lim √
x→−2 3+x−1
or say that it does not exist.

(A) 1
(B) ∞
(C) 0
(D) 2

6. At which point is the function f (x) = |x − 2| discontinuous?

(A) x = 0
(B) x = 2
(C) x = −2
(D) The function is continuous everywhere
1
7. What is the horizontal asymptote of the function h(x) = ?
x2 −1
(A) y = 0
(B) y = 1
(C) y = −1
(D) There is no horizontal asymptote
1
r
8. What is the vertical asymptote of the function f (x) = ?
x−2
(A) x = 0
(B) x = 1
(C) x = 2
(D) There is no vertical asymptote

9. For which values of a and b is f continuous everywhere?



2
ax − b,

 x < 2,
f (x) = 4, x = 2,

bx + 2,

x > 2.
1.7 Multiple Choice Questions 7

3
(A) a = 1, b =
4
5
(B) a = 1, b =
4
5
(C) a = , b = 1
4
3
(D) a = , b = 1
4
10. How to obtain the graph of f (x) given the graph of f (x + 3) − 2?

(A) Shift 2 units upward, and then 3 units to the right.


(B) Shift 2 units downward, and then 3 units to the right.
(C) Shift 2 units upward, and then 3 units to the left.
(D) Shift 2 units downward, and then 3 units to the left.

ANSWER: 1B 2C 3D 4A 5D 6D 7A 8C 9C 10A
8 FUNCTIONS AND LIMITS
Chapter 2

DERIVATIVES
2.1 Derivatives and Rates of Change
• The derivative of a function f at a number a is a number calculated by
f (a + h) − f (a)
f ′ (a) = lim (2.1)
h→0 h
if the limit exists.
f (a + h) − f (a) f (x) − f (a)
• f ′ (a) = limh→0 = limx→a .
h x−a
• If f ′ (a) exists, f is called differentiable at a.
• If a function is differentiable at a, it is continuous at a but the reverse statement is
not true (e.g., f (x) = |x| at 0).
• f ′ (a) is the slope of the tangent line of the curve y = f (x) at x = a and the line
tangent to the curve y = f (x) at x = a is given by

y = f ′ (a)(x − a) + f (a) (2.2)

• If s(t) is the position function of a particle at time t, then v(t) = s′ (t) and a(t) =
v ′ (t) = s′′ (t) are its velocity and acceleration at time t, respectively.

2.2 Differentiation Formulas


• Some basic formulas
d d α d √ 1 d 1 −1
 
(c) = 0, (x ) = αxα−1 , ( x) = √ , = ,
dx dx dx 2 x dx x x2
d x d 1 d d
(e ) = ex , (ln x) = , (sin x) = cos x, (cos x) = − sin x,
dx dx x dx dx

2.3 The Chain Rule


• If f is a function of u and u is a function of x, then we can define the composite
function f ◦ u. If f is differentiable at u(x) and u is differentiable at x, then we have
the
(f ◦ u)′ (x) = f ′ (u(x)) · u′ (x) (2.3)

or

df df du
= (2.4)
dx du dx

9
10 DERIVATIVES

• From the Chain Rule, we not that


d d du d
f (u) = [f (u)] = [f (u)]u′ (x)
dx du dx du

So, we have some useful formulas

d α d √ u′ (x) d 1 −u′ (x)


 
(u ) = αuα−1 u′ (x), ( u) = √ , = ,
dx dx 2 u dx u u2
d u d u′ (x) d
(e ) = eu u′ (x), (ln u) = , (sin u) = u′ (x) cos u.
dx dx u dx

2.4 Implicit Differentiation

• Suppose an implicit function y = y(x) is given by an equation F (x, y) = 0. To find


dy dy
, we differentiate both sides of F (x, y) = 0 with respect to x and then obtain
dx dx
using the Chain Rule.
dx
• For example, to find y ′ = where y = y(x) is the implicit function determined by
dy
x − 2yx + y = 1 (∗), we differentiate both sides of (∗) and obtain
2 3

d 2 d
(x − 3xy + y 3 ) = (1)
dx dx
2x − 3(x′ y + xy ′ ) + 3y 2 y ′ = 0
2x − 2y + y ′ (3y 2 − 2x) = 0
2y − 2x
y′ = 2 .
3y − 2x

2.5 Linear Approximations and Differentials

• The tangent line of the curve y = f (x) at x = a is

y = f ′ (a)(x − a) + f (a).

• The linear approximation of a function f (x) at x = a is

L(x) = f ′ (a)(x − a) + f (a).

For example, the linear approximation of f (x) = x3 − 2x at x = 2 is

L(x) = f ′ (2)(x − 2) + f (2)


= 10(x − 2) + 4 = 10x − 16.

• When x changes from x0 to x0 + ∆x, the corresponding chnage in f is

∆y = f (x0 + ∆x) − f (x0 ). (2.5)


2.6 Solved Problems 11

• If y = f (x), the differential dy is a function defined by

dy = f ′ (x) dx. (2.6)

• Differential dy can be used to estimate the change in f when there is a small change
in x. That is
∆y ≈ dy = f ′ (x) dx. (2.7)

2.6 Solved Problems


3x
1°. If f (x) = , find an equation of the tangent line to the curve y = f (x) at the
x2 −1
point (2, 2).
Solution. To find the equation of the tangent line, we first need to find the slope
of the tangent line, which is the derivative f ′ (x) evaluated at x = 2.
Given that f (x) = 3x
x2 −1
, we find the derivative f ′ (x) using the quotient rule

(x2 − 1) · (3) − (3x) · (2x)


f ′ (x) =
(x2 − 1)2
−3x2 − 3
= 2 .
(x − 1)2

Now, we evaluate f ′ (x) at x = 2 and the slope of the tangent line at (2, 2) is
5
f ′ (2) = − .
3
The equation of the tangent line at x = a is given by

y = f ′ (a)(x − a) + f (a)
5
= − (x − 2) + 2
3
5 16
=− x+ .
3 3

2°. Find the derivative of L(x) = ln(sin x).


Solution. Using the chain rule and the derivative of the natural logarithm function

1 d cos x
L′ (x) = · (sin x) = = cot x.
sin x dx sin x

dy
3°. Find if x2 + xy + y 2 = 1.
dx
12 DERIVATIVES

Solution.
d 2 d
(x + xy + y 2 ) = (1)
dx dx
dy dy
2x + y + x + 2y =0
dx dx
dy dy
x + 2y = −2x − y
dx dx
dy
(x + 2y) = −2x − y
dx
dy −2x − y
= .
dx x + 2y

4°. Find the derivative of the function y = cos(x sin x).


Solution. To find the derivative of the function y = cos(x sin x), we’ll apply the
chain rule.
Let u = x sin x, then y = cos u.
Now, we find the derivative of u with respect to x
du d
= (x sin x)
dx dx
= sin x + x cos x.

Now, applying the chain rule, we find the derivative of y with respect to x
dy dy du
= · .
dx du dx
dy
Given that y = cos u, we know that = − sin u. Therefore,
du
dy
= − sin u · (sin x + x cos x).
dx
Now, substitute u = x sin x, we obtain
dy
= − sin(x sin x) · (sin x + x cos x).
dx
So, the derivative of the function y = cos(x sin x) is
dy
= − sin(x sin x) · (sin x + x cos x)
dx

5°. Find the derivative of g(x) = sin( x).
Solution.
d √
g ′ (x) = (sin( x))
dx
√ d √
= cos( x) · ( x)
dx
√ 1
= cos( x) · √
2 x

cos( x)
= √ .
2 x
2.6 Solved Problems 13

6°. The radius of a sphere was measured and found to be 21 cm with a possible error
in measurement of at most 0.05 cm. What is the maximum error in using this value
of the radius to compute the volume of the sphere?
4
Solution. If the radius of the sphere is r, then its volume is V = πr3 . Denote by
3
dr = ∆r the error in the measured value of r. Then, the error in calculate V is ∆V,
which can be approximated by the differential

∆V ≈ dV = 4πr2 .

When r = 21, dr = ∆r = 0.05, we have

∆V ≈ 4π(21)2 0.05 ≈ 277.



7°. Use linear approximation to estimate 9.2.
Therefore, the maximum error in the calculated volume is about 277 cm3 .

Solution. Let f (x) = x and a = 9.

L(x) = f (a) + f ′ (a)(x − a)


√ 1
= 9 + √ (x − 9)
2 9
1
= 3 + (x − 9).
6
Using x = 9.2
1 91
L(9.2) = 3 + (9.2 − 9) = .
6 30

8°. A spotlight on the ground shines on a wall 12 m away. If a man 1.8 m tall walks
from the spotlight toward the building at a speed of 1.5 m/s, how fast is the length
of his shadow on the building decreasing when he is 4 m from the building?
Solution. Let x be the distance between the man and the building, and let y be the
length of his shadow on the building. We are given that dx
dt = −1.5 m/s (negative
because the man is walking towards the building) and we need to find dy
dt when x = 4
m.
We can use similar triangles to relate x, y, and the man’s height. Since the spotlight
is on the ground and the man is walking towards the building, the triangles formed
by the man, his shadow, and the spotlight are similar. Therefore, we have
y x
=
1.8 12
Rearranging, we get
5x
y=
3
Differentiating both sides with respect to time t, we get:

dy 5 dx
=
dt 3 dt
14 DERIVATIVES

dx
Substituting the given value of = −1.6 m/s, we get
dt
dy 5
= (−1.5) = −2.5 m/s
dt 3
Therefore, when the man is 4 m from the building, the length of his shadow on the
building is decreasing at a rate of 2.5 m/s.
9°. If a rock is thrown upward on the planet Mars with a velocity of 10 m/s, its height
(in meters) after t seconds is given by M (t) = 10t − 1.86t2 , t ≥ 0.
(a) When will the rock hit the surface?
(b) With what velocity will the rock hit the surface?
Solution. The rock hits the surface when its height is 0. So, we solve M (t) = 0
and get
10t − 1.86t2 = 0
or
t(10 − 1.86t) = 0.
This gives us two solutions t = 0 and 10 − 1.86t = 0.
Discarding t = 0, we solve 10 − 1.86t = 0 to find t.
10 − 1.86t = 0.
Hence,
10
t= ≈ 5.376.
1.86
So, the rock will hit the surface after approximately 5.376 seconds.
With what velocity will the rock hit the surface?
The velocity of the rock at time t is given by the derivative of M (t) with respect to
t. So, we find M ′ (t)
M ′ (t) = 10 − 2 × 1.86t = 10 − 3.72t
We evaluate M ′ (t) at t = 5.376
M ′ (5.376) = 10 − 3.72 × 5.376 ≈ −12.678.

Therefore, the velocity at which the rock hits the surface is approximately −12.678
m/s. Note that the negative sign indicates that the velocity is directed downwards.
√3
8+h−2
10°. The limit limh→0 represents the derivative of some function f at some
h
number a. State such an f and a.
Solution. To determine such an f and a, let’s rewrite the expression inside the
limit as
f (8 + h) − f (8)
lim .
h→0 h
Now, we need to find a function f (x) such that f (8) = 2. One such function is
√ d √ 1 2
f (x) = 3 x. Note that f is differentiable at x = 2 and f ′ (x) = 3
x = x− 3 .
dx 3

Therefore, the function f (x) = x and a = 8 satisfy the given limit representing
3

the derivative of f at a.
2.7 Supplementary Problems 15

2.7 Supplementary Problems


1°. If a rock is thrown upward on the planet Mars with a velocity of 10 m/s, its height
(in meters) after t seconds is given by M (t) = 10t − 1.86t2 , t ≥ 0.

(a) When will the rock hit the surface?


(b) With what velocity will the rock hit the surface?
3x
2°. If f (x) = , find an equation of the tangent line to the curve y = f (x) at the
x2 −1
point (2, 2).

3°. Express the limit as a derivative and evaluate.


π
 
cos + h − 0.5
3
lim .
h→0 h

4°. Find the derivative of the function y = cos(x sin x).


dy
5°. Find by implicit differentiation given that
dx

y cos x = x2 + y.

6°. A spotlight on the ground shines on a wall 12 m away. If a man 1.8 m tall walks
from the spotlight toward the building at a speed of 1.6 m/s, how fast is the length
of his shadow on the building decreasing when he is 4 m from the building?

7°. Find the linear approximation
√ of the function g(x) = 3
x − 1at a = 2 and use it to
approximate the numbers 1.2.
3

2.8 Multiple Choice Questions


1. What is the definition of the derivative of a function f (x) at a point x = a?
f (a + h)
(A) f ′ (a) = limh→0
h
f (a + h) − f (a)
(B) f ′ (a) = limh→0
h
f (x)
(C) f ′ (a) = limx→a
x
f (a)
(D) f ′ (a) = a

2. Which of the following represents the correct formula for the derivative of g(x) =
sin(3x)?

(A) g ′ (x) = 3 cos(x)


(B) g ′ (x) = cos(3x)
(C) g ′ (x) = 3 cos(3x)
(D) g ′ (x) = 3 cos(3x) sin(3x)
16 DERIVATIVES

5 √
3. Suppose f (u) = , u(x) = x + 3, what the derivative of f with respect to x
1+u2
when x = 1?
1
(A) −
5
1
(B)
5
2
(C) −
5
2
(D)
5
4. Compute dy and ∆y given y = x2 , x = 2, dx = ∆x = −0.2.

(A) dy = 0.04, ∆y = 0.036


(B) dy = −0.4, ∆y = −0.36
(C) dy = dy = 0.4, ∆y = 0.36
(D) dy = −0.8, ∆y = −0.76

5. Which of the following is the correct statement of the chain rule for differentiation?
d
(A) [f (g(x))] = f ′ (g(x))g ′ (x)
dx
d
(B) [f (g(x))] = f ′ (x)g ′ (x)
dx
d
(C) [f (g(x))] = f ′ (g(x))
dx
d
(D) [f (g(x))] = f (x)g(x)
dx
6. What is the slope of the tangent line to the curve y = −x3 + 2x + 5 at the point
(2, 1)?

(A) 14
(B) -10
(C) 1
(D) 11

7. Which of the following represents the linear approximation of f (x) = 1 + x at
x = 0?

(A) f (x) ≈ 1 + x
x
(B) f (x) ≈ 1 +
2
x
(C) f (x) ≈ 1 −
2
(D) f (x) ≈ 1 − x

8. What is the derivative of y 2 = 6x with respect to x?


dy 3
(A) =
dx y
2.8 Multiple Choice Questions 17

dy y
(B) =
dx 3
dy
(C) = 3y
dx
dy
(D) =3
dx
dy
9. Suppose x, y are functions of t such that x2 y + y 3 = 5. When t = 3, find if
dt
dx
= 3 and x = 2.
dt
4
(A) −
7
4
(B) −
7
12
(C)
7
12
(D) −
7
10. What is the derivative of y with respect to x given y 3 + x2 = 2xy?
dy 2y − 2x
(A) =
dx 3y 2 − 2x
dy 2x − 2y
(B) = 2
dx 3y − 2x
dy 2x − 2y
(C) = 2
dx 3y + 2x
dy 2y − 2x
(D) = 2
dx 3y + 2x

ANSWER: 1B 2C 3A 4D 5A 6B 7B 8A 9D 10A
18 DERIVATIVES
Chapter 3

APPLICATIONS OF
DIFFERENTIATION
3.1 Maximum and Minimum Values

Suppose f (x) is a function defined on D.

• If there exists a number x0 ∈ D such that for δ > 0 small and for all x ∈ (x0 −δ, x0 +δ)
we have f (x0 ) ≤ f (x), then we say that f has a local minimum at x0 and f (x0 ) is
a local minimum value of f .

• If f (x0 ) ≤ f (x) for all x ∈ D, the we say that f attains its absolute minimum at x0
and f (x0 ) is its absolute minimum value.

• If f (x0 ) ≥ f (x) for all x ∈ (x0 − δ, x0 + δ), the we say that f has a local maximum
at x0 and f (x0 ) is a local maximum value of f .

• If f (x0 ) ≥ f (x) for all x ∈ D, the we say that f attains its absolute maximum at x0
and f (x0 ) is its absolute maximum value.

• If f ′ (c) = 0 and f ′ changes sign at c, then f (c) is a local minimum/maximum value.

• If f has a local maximu/minimum value at c, then f ′ (c) = 0 of f ′ (c) does not exist.

• If a function f is continuous on a closed interval [a, b], it attains its absolute maxi-
mum and minimum values.

• There are functions that have no maximum or minimum.

3.2 Mean Value Theorem

• Suppose f is a function such that:

(i) f is continuous on [a, b],


(ii) f is differential on (a, b).

Then there exists a number c ∈ (a, b) such that

f (b) − f (a)
f ′ (c) = (3.1)
b−a

• (Rolle’s Theorem) Suppose f is a function such that

(i) f is continuous on [a, b],


(ii) f is differential on (a, b),

19
20 APPLICATIONS OF DIFFERENTIATION

(iii) f (a) = f (b).

Then there exists a number c ∈ (a, b) such that f ′ (c) = 0.

3.3 Optimization Problems

• Optimization problems are problems in real life that involve finding maximum or
minimum value of a function of one variable f (x) over a certain interval [a, b], where
a and b are real numbers and a < b.

• To find the maximum or minimum value of the function f (x) over the interval [a, b],
we follow the following steps:
1. Determine the critical points of f (x) in the interval (a, b). These are the points
where the derivative of f (x) is zero or undefined.
2. Evaluate the function f (x) at the critical points and at the endpoints a and b.
3. Compare the values of f (x) at the critical points and endpoints to determine the
maximum and minimum values of f (x) over the interval [a, b].
Example. Consider the function f (x) = x2 − 4x + 5 defined over the interval [0, 3].
1. The critical point of f (x) occurs where its derivative f ′ (x) = 2x − 4 is zero:

2x − 4 = 0 =⇒ x = 2

So, x = 2 is the critical point.


2. Evaluate f (x) at x = 0, 2, and 3

f (0) = 5, f (2) = 1, f (3) = 2

3. Since f (2) = 1 is less than f (0) = 5 and f (3) = 2, the minimum value of f (x)
over the interval [0, 3] is f (2) = 1.

3.4 Newton’s Method

• Newton’s method is used to appriximate the root of an equation f (x) = 0. The


method consists of two steps.
Step 1. Choose an initial guess x1 near the root of the equation.
.
Step 2. Compute x for n = 1, 2, .. using the iterative formula
n

f (xn )
xn+1 = xn − (3.2)
f ′ (xn )

Example. Consider finding the root of the function f (x) = x3 − 2x2 − 5 using
Newton’s method.
1. Choose an initial guess x1 . Let’s choose x1 = 2.
3.5 Antiderivatives 21

2. Compute f (x1 ) and f ′ (x1 )

f (2) = (2)3 − 2(2)2 − 5 = 1

f ′ (2) = 3(2)2 − 4(2) = 4

3. Apply (3.2) to obtain the next approximation x2 :


1 7
x2 = 2 − =
4 4

4. Repeat the process until the desired level of accuracy is achieved or until the
maximum number of iterations is reached.

3.5 Antiderivatives
• If a function F satisfies F ′ (x) = f (x), then F is called an antiderivative of the
function f .

• If F is an antiderivative of f , then F + C is also an antiderivative of f , where C is


a constant.

3.6 Solved Problems


1°. Find the maximum and minimum values of f (x) = x3 − 3x2 + 2 on the interval
[−1, 3].
Solution. To find the critical points, we find where the derivative is zero

f ′ (x) = 3x2 − 6x = 3x(x − 2) = 0,

we obtain the critical values x = 0 and x = 2.


Now we calculate the values of f at the critical points and endpoints

f (−1) = 6, f (0) = 2, f (2) = 2, f (3) = −1

So, the maximum value is 6 and the minimum value is −1.

2°. Suppose f is a function such that f ′ (x) ≥ 3 for all x and f (1) = 7, how small can
f (5) possibly be?
Solution. According to the Mean Value Theorem, if f is continuous on [a, b] and
differentiable on (a, b), then there exists a point c in (a, b) such that
f (b) − f (a)
f ′ (c) =
b−a

Given that f ′ (x) ≥ 3 for all x, we know that f ′ (c) ≥ 3.


Applying the Mean Value Theorem to the interval [1, 5], there exists a number
c ∈ (1, 5) such that
f (5) − f (1)
f ′ (c) =
5−1
22 APPLICATIONS OF DIFFERENTIATION

Since f ′ (c) ≥ 3, we have


f (5) − f (1)
≥3
4
f (5) − 7 ≥ 12
f (5) ≥ 10

So, the smallest possible value for f (5) is 19.

3°. Suppose f is an odd function and is differentiable everywhere. Prove that for every
positive number b, there exists a number c ∈ (−b, b) such that f ′ (c) = f (b)
b .
Solution. Since f is an odd function, f (−x) = −f (x) for all x. Also, since f is
differentiable everywhere, it is continuous on any closed interval [−b, b] and differ-
entiable on the open interval (−b, b) for every positive number b.
According to the Mean Value Theorem (MVT), there exists a point c in (−b, b) such
that:
f (b) − f (−b)
f ′ (c) =
b − (−b)

Given that f is an odd function, f (−b) = −f (b). Therefore, we rewrite the above
equation as:
f (b) + f (b) 2f (b) f (b)
f ′ (c) = = .
b − (−b) 2b b

4°. Find the dimensions of a rectangle with perimeter 100 m whose area is as large as
possible.
Solution. The area A of a rectangle is given by A = lw, where l is the length and
w is the width, and the perimeter P is given by P = 2l + 2w.
Given that the perimeter P is 100 m, we have the equation

2l + 2w = 100

l = 50 − w

Now, we can express the area A in terms of one variable

A = l · w = (50 − w) · w = 50w − w2

To find the maximum area, we take the derivative of A with respect to w and set it
equal to zero
dA
= 50 − 2w = 0
dw
w = 25

So, the width of the rectangle is 25 m. Substituting this value back into the equation
for the perimeter, we find the length

l = 50 − w = 25

Therefore, the dimensions of the rectangle with perimeter 100 m whose area is as
large as possible are 25 m by 25 m.
3.6 Solved Problems 23

5°. Find the point on the line y = 2x + 3 that is closest to the origin.
Solution. To find the point on the line y = 2x + 3 that is closest to the origin,
we need to minimize the distance between the origin (0, 0) and a point (x, y) on the
line.
For the point (x, y) on the line y = 2x + 3, the is
q
d= x2 + (2x + 3)2

To minimize d, we minimize d2 , since d2 is easier to differentiate

d2 = x2 + (2x + 3)2 =: f (x)

Now, we differentiate f with respect to x

f ′ (x) = 10x + 12

Setting f ′ (x) equal to zero to find the critical point

10x + 12 = 0
6
x=− .
5
Now, we find the corresponding y value using the equation of the line y = 2x + 3
and obtain y = 35 .
 
So, the point on the line y = 2x + 3 that is closest to the origin is − 65 , 35 .

6°. A box with a square base and open top must have a volume of 32, 000cm3 . Find the
dimensions of the box that mini mize the amount of material used.
Solution. We need to minimize the surface area of the box, which consists of the
area of the square base and the area of four sides.
Let x be the length of each side of the square base, and let h be the height of the
box.
The volume V of the box is given by

V = x2 · h = 32000

and then
32000
h= .
x2

The area of the square base is x2 , and the combined area of the four sides is 4xh.
So, the surface area is
A = x2 + 4xh.

Hence, we get
32000 128000
 
A = x2 + 4x = x2 + .
x2 x
24 APPLICATIONS OF DIFFERENTIATION

Let’s differentiate A = A(x) with respect to x

128000
A′ (x) = 2x − .
x2

Setting A′ (x) equal to zero to find the critical point

128000
2x − =0
x2

x3 = 64000
x = 40.
Hence, we also obtain
h = 20.

7°. Use Newton’s method with x1 = 1 to find x3 , the third approximation to the solution
of the equation x3 − x = 1.
Solution. We follow the iterative formula

f (xn )
xn+1 = xn −
f ′ (xn )

where f (x) = x3 − x − 1 and f ′ (x) is the derivative of f (x).


First, let’s find f ′ (x)
f (x) = x3 − x − 1 = 3x2 − 1.

Now, let’s plug in x1 = 1 to find x2

f (x1 )
x2 = x1 −
f ′ (x1 )

(13 − 1 − 1)
x2 = 1 − = 1.5.
(3 · 12 − 1)

Now, let’s plug in x2 to find x3

f (x2 )
x3 = x2 −
f ′ (x2 )

(1.53 − 1.5 − 1)
x3 = 1.5 − ≈ 1.348.
(3 · 1.52 − 1)

Thus, the third approximation to the solution of the equation is x3 ≈ 1.348.

8°. Find the numbers c in the Rolle’s theorem for the function f (x) = x3 − 2x2 − 7x.
Solution. Suppose that a function f statisfies the following
1. f (x) is continuous on the closed interval [a, b].
2. f (x) is differentiable on the open interval (a, b).
3. f (a) = f (b).
3.6 Solved Problems 25

Then, there exists a number c ∈ (a, b) such that

f ′ (c) = 0.

Notice that the function f (x) = x3 − 2x2 − 7x is differentiable everywhere. The


derivative of f is
f ′ (x) = 3x2 − 4x − 7.

By setting f ′ (x) = 0, we obtain

3x2 − 4x − 7 = 0
7
x = −1, x = .
3

Therefore, the numbers c in Rolle’s theorem for the function f (x) = x3 − 2x2 − 7x
are c = −1 and c = 37 .

9°. Given the graph of f ′ .

(a) On what intervals is f increasing? Explain.


(b) At what values of x does f have a local maximum or minimum? Explain
(c) On what intervals is f concave upward or concave downward? Explain.
(d) What are the x-coordinates of the inflection points of f ? Why?
Solution. (a) If f ′ (x) ≥ 0 for x ∈ I, then f is increasing on the interval I.
From the graph of f ′ , we have the intervals where f ′ ≥ 0 (above the x-axis) are

((2, 4), (6, 9).

Therefore, f is increasing on (2, 4) and on (6, 9).


(b) If f ′ (c) = 0 and f ′ changes sign at c, then f (c) is a local maximum/minimum of
the function f .
From the graph of f ′ , we have f ′ (2) = 0 and f ′ changes sign from negative to positive
at x = 2 from left to right. Therefore, f changes from decreasing to increasing at 2
from left to right and hence f (2) is a local minimum of the function f .
Similarly, f (4) is a local maximum, and f (6) is a local minimum.
(c) f is concave upward (downward) on a interval I if f ′ is increasing (decreasing,
respectively) on I.
From the graph of f ′ , we see that f ′ is increasing on the intervals (1, 3), (5, 7), (8, 9).
Therefore, f is concave upward on each of these intervals.
26 APPLICATIONS OF DIFFERENTIATION

Similarly, f is concave downward on each of these intervals

(0, 1), (3, 5), (7, 8).

(d) If f changes from concave upward (downward) to concave downward (upward,


respectively) at the point (c, f (c)), then it is called an inflection point of f .
From the graph and part (c), we see that the x-coordinates of the inflection points
are
1, 3, 5, 7, and 8.

3.7 Supplementary Problems

1°. Find the maximum and minimum values of f (x) = cos x + x


2 on the interval [0, π2 ].

2°. Suppose f is a function such that f ′ (x) ≤ 5 for all x and f (1) = 5. Show that for
every b > 1,
f (b) ≤ 5b.

3°. Find the point on the line y = x + 2 that is closest to the point (1, 0).

4°. Use Newton’s method with x1 = 1 to find x3 , the third approximation to the solution
of the equation x5 − 7x = 3.
2
5°. Find the numbers c in the Rolle’s theorem for the function f (x) = ex − x2 .

6°. Given the graph of f ′ .

(a) On what intervals is f increasing? Explain.


(b) At what values of x does f have a local maximum or minimum? Explain
(c) On what intervals is f concave upward or concave downward? Explain.
(d) What are the x-coordinates of the inflection points of f ? Why?

7°. A boat leaves a dock at 1:00 PM and travels due south at a speed of 18 km/h.
Another boat has been heading due east at 15 km/h and reaches the same dock at
3:00 PM. At what time were the two boats closest together?

8°. An algorithm called gradient descent can be used to find a local minimum value
of a function f . This algorithm is described as follow
- Step 1. Choose an initial value a0 and a learning rate γ > 0
3.8 Multiple Choice Questions 27

- Step 2. Repeat computing

an+1 = an − γf ′ (an )

until we meet the stopping condition.


For good learning rate, the sequence {f (an )} tends to a local minimum value of f .
Given f (x) = x3 − 3x + 5. Find a1 , a2 , a3 , a4 when
(a) a0 = 0, λ = 0.1
(b) a0 = 2, λ = 0.5

3.8 Multiple Choice Questions

1. Suppose f is a differentiable function on (a, b). What condition must be satisfied by


f (x) at a point c ∈ (a, b) for it to have a local maximum or minimum at that point?

(A) f ′ (c) = 0
(B) f ′′ (c) = 0
(C) f ′ (c) > 0
(D) f ′′ (c) > 0

2. The Mean Value Theorem states that for a function f (x) continuous on the closed
interval [a, b] and differentiable on the open interval (a, b), there exists at least one
point c in (a, b) such that:
f (b)−f (a)
(A) f ′ (c) = b−a
f ′ (b)−f ′ (a)
(B) f (c) = b−a
(C) f ′ (c) = f (b) − f (a)
(D) f (c) = f ′ (b) − f ′ (a)

3. Newton’s method is an iterative numerical method used to find:

(A) the maximum value of a function


(B) the minimum value of a function
(C) the root of a function
(D) the antiderivative of a function

4. Which of the following represents Newton’s method for finding the root of a function
f (x) = 0?
f (xn )
(A) xn+1 = xn − f ′ (xn )
(B) xn+1 = xn − f (xn )
f ′ (xn )
(C) xn+1 = xn − f (xn )
f (xn )
(D) xn+1 = xn + f ′ (xn )
28 APPLICATIONS OF DIFFERENTIATION

5. Consider the function f (x) = x3 − 2x2 . Which of the following statements is/are
true?
(i) f is increasing on (0, 1)
(ii) f is concave up on ( 32 , +∞)
(A) Only (i)
(B) Only (ii)
(C) Both (i) and (ii)
(D) Neither (i) nor (ii)
6. Which of the following conditions is sufficient for f to have a local minimum at
x = c?
(A) f ′ (c) = 0 and f ′′ (c) > 0
(B) f ′ (c) = 0 and f ′′ (c) < 0
(C) f ′ (c) > 0 and f ′′ (c) = 0
(D) f ′ (c) < 0 and f ′′ (c) > 0
7. Find the number c < 0 in the Rolle’s theorem for the function f (x) = x3 − x2 − x?
(A) c = − 12
(B) c = −2
(C) c = − 13
(D) c = −1
8. Which of the following represents the optimization problem of finding the maximum
area A of a rectangle with sides x, y and fixed perimeter P ?
(A) Minimize P = 2x + 2y
(B) Maximize A = xy
(C) Minimize A = xy
(D) Maximize P = xy
9. What is the value of c guaranteed by the Mean Value Theorem for the function

f (x) = x on the interval [1, 4]?
(A) c = 23
(B) c = 94
(C) c=2
(D) c=3
10. Find the point on the line y = x − 3 closet to the origin O.
(A) (0, −3)
 
(B) − 23 , 23
 
(C) 3 3
2, −2
(D) (3, 0)

ANSWER: 1A 2A 3C 4A 5B 5A 6A 7C 8B 9B 10C
Chapter 4

INTEGRALS
4.1 The Definite Integral
• Suppose f is a function defined for x ∈ [a, b]. Divide [a, b] into n equal subintervals
[xi , xi+1 ] with width ∆x = b−an and a = x0 ≤ x1 ≤ · · · ≤ xn = b. The definite
integral of f from a to b is defined by
Z b n
f (x) dx = lim f (x∗i )∆x (4.1)
X
a n→∞
i=1

provided this limit exists and doesn’t depend on the choice of sample points x∗i ∈
[xi−1 , xi ].
• If the limit exists, f is called integrable on [a, b].
• The sum n
f (x∗i )∆x = f (x∗1 )∆x + f (x∗2 )∆x + · · · + f (x∗n )∆x
X

i=1

is called a Riemann sum.


• Suppose we want to approximate the area under the curve of the function f (x) = x2
over the interval [0, 2] using Riemann sums.
Let’s divide the interval [0, 2] into 4 subintervals of equal width. The width of each
subinterval is given by ∆x = 2−0 n = 2.
1

We’ll use the right endpoint of each subinterval to sample the function. The right
endpoints are x1 = 12 , x2 = 1, x3 = 23 , and x4 = 2.
 2
The height of the rectangle corresponding to the i-th subinterval is f (xi ) = i
2 =
i2
4.
The Riemann sum is the sum of the areas of all rectangles

12 22 32 42
R4 = f (x1 )∆x + f (x2 )∆x + f (x3 )∆x + f (x4 )∆x = + + + .
8 8 8 8
You can compute R4 to estimate the area under the curve of f (x) = x2 over the
interval [0, 2].
• If f (x) ≥ for x ∈ [a, b], then ab f (x) dx equals the area of the region under the curve
R

y = f (x) and over the interval [a, b] (see Figure 4.2).


• Suppose tha a function f is continuous on a interval [a, b]. Then there exists a
number c ∈ [a, b] such that
Z b
f (x) dx
f (c) = a
. (4.2)
b−a
The value f (c) is called the average value of f over [1, b].

29
30 INTEGRALS

Fig. 4.1: If f (x) ⩾ 0, Riemann sum is the sum of areas of rectangles

Rb
Fig. 4.2: If f (x) ⩾ 0, integral a
f (x) dx is the area under the curve y = f (x) from a to b.

4.2 The Fundamental Theorems of Calculus


• If f is continuous on [a, b], then the function g defined by
Z x
g(x) = f (t) dt, x ∈ [a, b]
a

is continuous on [a, b] and differentiable on (a, b). Moreover,


Z x
d


g (x) = f (t) dt = f (x) (4.3)
dx a

• In general, we have
Z b(x) !
d
f (t) dt = f (b(x))b′ (x) − f (a(x))a′ (x) (4.4)
dx a(x)

• If f is continuous on [a, b], then


Z b
f (x) dx = F (b) − F (a), (4.5)
a

where F is an antiderivative of f , that is, F ′ = f .


• (Net change theorem) The integral of the rate of change is the net change, that is,
Z b
f ′ (x) dx = f (b) − f (a) (4.6)
a
Z
• From FTC, an indefinite integral f (x) dx is a family of functions F (x) + C, where
F ′ (x) = f (x).
• If v(t) is the velocity function of a particle moving in a straight line, the the distance
traveled during the time period [a, b] is given by
Z b
Distance traveled = |v(t)| dt (4.7)
a
4.3 The Substitution Rule 31

4.3 The Substitution Rule

• If u = g(x) is a differentiable function and f (.) is continuous on the range of g.


Then,

Z b Z g(b)
f (g(x))g ′ (x) dx = f (u) du (4.8)
a g(a)

x
Example. Let’s evaluate the integral e√x dx using the substitution rule.
R


1. We choose the substitution u = x, so du = 2√1 x dx.
2. We rewrite the integral in terms of u:

e x
Z Z
√ dx = 2eu du
x

3. We can now evaluate the integral with respect to u:


Z
2eu du = 2eu + C


4. Finally, we substitute back u = x to get the answer in terms of x and obtain
that

e x √
Z
√ dx = 2e x + C,
x

where C is the constant of integration.

4.4 Solved Problems

1°. Find the definite integral of f (x) = 3x2 − 2x from x = 1 to x = 3.


Solution.
Z 3   3
(3x2 − 2x) dx = x3 − x2
1 1
= (3 − 3 ) − (13 − 12 ) = 18.
3 2

ex
2°. Evaluate the indefinite integral
R
1+ex dx.
Solution.
ex
To evaluate the indefinite integral dx, we use the substitution
R
1+ex

u = 1 + ex ,

du = ex dx.

Substituting u = 1 + ex and du = ex dx into the integral, we get


32 INTEGRALS

ex 1
Z Z
dx = du
1 + ex u

Now, integrating 1
u with respect to u, we obtain

1
Z
du = ln |u| + C
u
Substituting back u = 1 + ex and C for the constant of integration, the indefinite
integral becomes

ex
Z
dx = ln |1 + ex | + C.
1 + ex
3°. Find the Riemann sum from a = 1 to b = 2 of the function f (x) = x+1 ,
x
with n = 4
and taking left endpoints as sample points.
Solution. To find the Riemann sum of the function f (x) = x+1 x
from a = 1 to
b = 2, using left endpoints as sample points and n = 4, we first need to calculate
the width of each subinterval

b−a 2−1 1
∆x = = = .
n 4 4
Now, we compute the left endpoints of the subintervals

1 5 5 1 3 3 1 7
x1 = 1, x2 = 1 + ∆x = 1 + = , x3 = + = , x4 = + = .
4 4 4 4 2 2 4 4

Next, we evaluate f (x) at these left endpoints

1 5 5/4 5 3 3/2 3 7 7/4 7


     
f (1) = , f = = , f = = , f = = .
2 4 5/4 + 1 9 2 3/2 + 1 5 4 7/4 + 1 11

Now, we calculate the Riemann sum

3 5 7
      
R4 = ∆x f (1) + f +f +f
2 4 4
1 1 5 3 7 19
 
= + + + = .
4 2 9 5 11 36

Therefore, the Riemann sum of the function f (x) = x+1


x
from a = 1 to b = 2, using
left endpoints as sample points and n = 4, is 36 .
19


4°. Find the area under the curve y = 1 + x and over the interval [0, 3].

Solution. To find the area under the curve y = 1 + x and over the interval [0, 3],
we integrate the function y with respect to x
4.4 Solved Problems 33

Z 3√
A= 1 + x dx
0

Using the substitution u = 1 + x, we have du = dx, and when x = 0, u = 1 and


when x = 3, u = 4. The integral becomes
Z 4

A= u du
1

Now, integrating with respect to u


4
2
A = u3/2
3
1
14
= .
3

So, the area under the curve y = 1 + x and over the interval [0, 3] is 3 .
14

5°. Find the derivative with respect to x the function

ßt2
Z x
sin( ) dt.
0 2

Solution. The Fundamental Theorem of Calculus states that if


Z x
F (x) = f (t) dt,
a

where f is continuous on [a, b], then


Z x
d

F ′ (x) = f (t) dt = f (x).
dx a

Rx 2
Here, our function is F (x) = 0 sin( πt2 ) dt, and we want to find F ′ (x).
According to the Fundamental Theorem of Calculus, we have
!
πt2 πx2
Z x
′ d
F (x) = sin( ) dt = sin( ).
dx 0 2 2

6°. Find the derivative with respect to x the function


Z √x
(1 + θ2 ) dθ.
x

Solution. To find the derivative with respect to x of the function


Z √x
(1 + θ2 ) dθ,
x

we apply the Leibniz Rule, which is a generalization of the Fundamental Theorem


of Calculus for definite integrals with variable limits of integration.
34 INTEGRALS

R b(x)
The Leibniz Rule states that if F (x) = a(x) f (t) dt, where f is continuous and a(x)
and b(x) are continuously differentiable, then
Z b(x) !
d
f (t) dt = f (x, b(x))b′ (x) − f (x, a(x))a′ (x).
dx a(x)

Here, our function is


Z √x
F (x) = (1 + θ2 ) dθ.
x

and

a(x) = x, b(x) = x.

Differentiate a(x) and b(x) with respect to x, we obtain

1
a′ (x) = 1, b′ (x) = √ .
2 x

Apply the Leibniz Rule, we have


1
F ′ (x) = (1 + x) · √ − (1 + x2 ).
2 x
1+x
= √ − 1 − x2 .
2 x

Thus,
Z √x
1+x
!
d
(1 + θ ) dθ 2
= √ − 1 − x2 .
dx x 2 x

7°. Find the indefinite integral x cos(x2 ) dx.


R

Solution. To find the indefinite integral x cos(x2 ) dx, we use the substitution
R

u = x2 .

Then
du
du = 2x dx or dx = .
2x

Substituting u = x2 and dx = du
2x into the integral, we get

du 1
Z Z Z
x cos(x2 ) dx = cos(u) · = cos(u) du.
2 2
We then obtain

1
Z
cos(u) du = sin(u) + C
2

Substituting back u = x2 , the indefinite integral becomes

1
Z
x cos(x2 ) dx = sin(x2 ) + C.
2
4.5 Supplementary Problems 35

8°. Use the substitution u = x2 + 1 to evaluate the integral

x3
Z 1
√ dx.
0 1 + x2

Solution. Let u = x2 + 1, then du = 2x dx and

du
x3 dx = x2 · x dx = (u − 1)
2

To find the new limits of integration we note that

x = 0, u = 1, x = 1, u = 2.

Now, integrate with respect to u

x3 u−1
Z 1 Z 2
√ dx = √ du
0 1 + x2 1 2 u
1 2 1/2
Z
= (u − u−1/2 ) du
2 1

1 2 3/2 2− 2
  2
= u − 2u1/2
= .
2 3 1 3

R1 3

So, the value of the integral 0
√x
1+x2
dx is 3 .
2− 2


9°. The linear density of a rod of length 1 m is given by ρ(x) = 4 + 2 x measured in
kilograms per meter, where is measured in meters from one end of the rod. Find
the total mass of the rod.
Solution. To find the total mass of the rod, we need to integrate the linear density
function ρ(x) over the length of the rod, which is from x = 0 to x = 1.
The total mass M of the rod is given by the integral
Z 1
M= ρ(x) dx
0
Z 1
√ Z 1 Z 1

= (4 + 2 x) dx = 4 dx + 2 x dx
0 0 0
2 14
=4+ =
3 3

So, the total mass of the rod is 14


3 kilograms.

4.5 Supplementary Problems

1°. Find the Riemann sum from a = 2 to b = 4 of the function f (x) = x2 + −2x, with
n = 4 and taking right endpoints as sample points.

2°. Find the area under the curve y = x 3x2 + 1 and over the interval [0, 1].
36 INTEGRALS

3°. Suppose f is a continuous function such that


Z √x
f (t) dt = x2 − 3x, x > 1.
1

Find the value of f (4).

4°. If Z √x p
f (x) = 5 + t2 dt
1
and Z y
g(y) = f (x) dx.
2
Find g ′′ (4).

5°. Find the derivative with respect to y the function


Z y2 p
1 + η dη.
1
Rx
6°. Let g(x) = 0 f (t) dt, where f is the function whose graph are shown below.

(a) Evaluate g(x) for x = 0, 2, and 6.


(b) Find g ′ (2), g ′ (3).
(c) Where does g have a local maximum value?
R ln2 (3x)
7°. Find the indefinite integral x dx.

8°. Evaluate the integral Z 4 p


x3 9 + x2 dx.
0

9°. The linear density of a rod of length 4 m is given by ρ(x) = 9 + 2 x measured in
kilograms per meter, where is measured in meters from one end of the rod. Find
the total mass of the rod.

4.6 Multiple Choice Questions


1. What is the definite integral of f (x) = 3x2 from x = 0 to x = 2?

(A) 4
4.6 Multiple Choice Questions 37

(B) 6
(C) 8
(D) 10
Rx
2. Suppose 0 f (t) dt = x2 − 3x. What is the value of f (3)?

(A) 2
(B) 3
(C) 4
(D) 5

3. What is the Riemann of the function f (x) = 3x2 from a = 0 to b = 2 with n = 4


using left endpoints as sample points?

(A) 15
4
(B) 17
4
(C) 19
4
(D)* 21
4

4. Which of the following represents the fundamental theorem of calculus?


f (b)−f (a)
(A) f ′ (c) = b−a
d Rx
(B) dx a f (t) dt = f (x)
f (a+h)−f (a)
(C) f ′ (a) = limh→0 h
Rb
(D) f (x) dx = limn→∞ ni=1 f (xi )∆x
P
a
Rx 2
5. Find the derivative of g(x) = 1 (t + 3) dt

(A) g ′ (x) = 2x
(B) g ′ (x) = x2 + 3
(C) g ′ (x) = 2x(x2 + 3)
(D) g ′ (x) = 2x + 3

d R r2

6. Find dr 3 x + 1 dx

(A) 2r r2 + 1
√ √
(B) 2r r2 + 1 − 3 r2 + 1
√ √
(C) 2r r + 1 − 3 r + 1

(D) 2r r + 1
Rx
7. What is the derivative of the function f (x) = 0 cos(t2 ) dt?

(A) cos(x2 )
(B) − sin(x2 )
(C) sin(x2 )
38 INTEGRALS

(D) − cos(x2 )

(A) sin(x2 ))
(B) cos x
(C) 2 sin x cos x
(D) sin x

8. Which substitution is appropriate for the integral 1


R
x ln(x) dx?

(A) u = ln(x)
(B) u = 1
x
(C) u = x
(D) u = 1
ln x
R sin(x)
9. Which substitution is appropriate for the integral cos2 (x)
dx?

(A) u = sin(x)
(B) u = cos(x)
(C) u = tan(x)
(D) u = sec(x) = 1
cos2 (x)

10. Which substitution is appropriate for the integral x x2 + 1 dx?
R

(A) u = x

(B) u = x2 + 1
(C) u = x2
(D) u = x + 1

11. Suppose ϕ is a function such that ϕ(1) = −3, ϕ(3) = 7, ϕ′ (1) =R 5, ϕ′ (3) =
4, ϕ′′ (1) = 13, ϕ′′ (3) = 8, and ϕ′′ is continuous everywhere. Evaluate 13 ϕ′′ (x) dx.

(A) 5
(B)* −1
(C) 10
(D) −5

12. Suppose the velocity function (in m/s) of a particle moving along a line is v(t) =
t − 2, t ∈ [0, 3]. Find the distance traveled during the given time interval.

(A) 2
(B) 2.5
(C) 3
(D) 3.5
Rx
13. The following
Rx
figures shows the graph of f, f ′ , and 0 f (t) dt. Which one is the
graph of 0 f (t) dt?
4.6 Multiple Choice Questions 39

(A) a
(B) b
(C) c
(D) Not enough information to determine

14. What does the mean value theorem for integrals state?

(A) If f (x) is continuous on [a, b], then there exists a c in (a, b) such that f (c) =
1 Rb
b−a a (x) dx
f
(B) If f (x) is differentiable on [a, b], then there exists a c in (a, b) such that f ′ (c) =
1 Rb
b−a a f (x) dx
(C) RIf f (x) is continuous on [a, b], then there exists a c in (a, b) such that f (c) =
b
a f (x) dx
(D) RIf f (x) is differentiable on [a, b], then there exists a c in (a, b) such that f (c) =
b ′
a f (x) dx

ANSWER: 1C 2B 3D 4B 5B 6A 7A 8A 9B 10B 11B 12B 13A 14A


40 INTEGRALS
Chapter 5

TECHNIQUES OF INTEGRATION

5.1 Integration by Parts

• From the formula (uv)′ = u′ v + v ′ u, we have


Z Z
u dv = uv − v du (5.1)

and Z b Z b
udv = (uv)|ba − v du (5.2)
a a

where u and v are differentiable functions of x.

• We usually let u = u(x) be such that du = u′ (x) dx is rather simple compare to u(x)
in the original integral. For example, let’s evaluate the integral
Z
x ln x dx

using integration by parts.


Here, we choose
u = ln x and dv = x dx

Then, we differentiate u to get du and integrate dv to get v

1 x2
du = dx and v=
x 2

Now, apply (5.1) yields

x2 x2 1
Z Z Z
x ln x dx = uv − v du = ln x − dx
2 2 x

x2 x x2 x2
Z
= ln x − dx = ln x − + C,
2 2 2 4
where C is the constant of integration.

5.2 Approximate Integration

If f is continuous on [a, b] and a = x0 ≤ x1 ≤ · · · ≤ xn = b with equal subintervals


[xi−1 , xi ] and ∆x = xi − xi−1 = b−a
n , i = 1, 2, ..., n.

41
42 TECHNIQUES OF INTEGRATION

• Midpoint Rule
Z b n
f (x) dx ≈ f (xi )∆x, (5.3)
X
a i=1
xi−1 +xi
where xi = 2 .

• Trapezoidal Rule

∆x
Z b
f (x) dx ≈ (f (x0 ) + 2f (x1 ) + · · · + 2f (xn−1 ) + f (xn )) . (5.4)
a 2

• Simpson’s Rule

∆x
Z b
f (x) dx ≈ [f (x0 ) + 4f (x1 ) + 2f (x2 ) + 4f (x3 ) + · · ·
a 3

+2f (xn−2 ) + 4f (xn−1 ) + f (xn )],


where n is even.

• Simpson’s rule gives good approximation even when n is small.

5.3 Improper Integrals


R∞ Ra R∞
• Integrals in one of the forms a f (x) dx, −∞ f (x) dx and −∞ f (x) dx are called
Type I improper integrals.
R∞ Rt
• We call the integral a f (x) dx convergent if limt→∞ a f (x) dx is a number.
Otherwise, we say that it diverges.
Ra Ra
• −∞ f (x) = limt→−∞ t f (x) dx
R∞ R∞ Ra
• The integral −∞ f (x) is convergent if both integrals a f (x) dx and −∞ f (x) dx
converge.
Example. Consider the improper integrals
1 1
Z ∞ Z ∞
dx and dx
1 x +1
4
1 x3

Let’s compare these integrals using the comparison principle.


1. For x ≥ 1, we have 0 ≤ 1
x4 +1
≤ x13 .
2. The improper integral of x13 converges since:

1 1 1 1 1 1
Z ∞ Z b  b  
dx = lim dx = lim − = lim − + =
1 x3 b→∞ 1 x3 b→∞ 2x2 1 b→∞ 2b2 2 2

3. By the comparison principle, since 0 ≤ x41+1 ≤ x13 and the improper integral of
1
x3
converges, the improper integral of x41+1 also converges.
5.4 Solved Problems 43

• Type II Improper Integrals.


Z b
f (x) dx,
a

where the interval [a, b] contains the numbers at which the function f is infinitely
discontinuous.
An example is the Type II improper integral

1
Z 1
√ dx
0 x

This integral represents the area under the curve y = √1


x
from x = 0 to x = 1. The
function √1
x
has an infinite discontinuity at x = 0. We can evaluate this integral
using the limit
Z 1
1
Z 1
1  √ 1 √
√ dx = lim √ dx = lim 2 x b = lim (2 − 2 b) = 2
0 x b→0+ b x b→0+ b→0+

So, the integral converges and its value is 2.

5.4 Solved Problems

1°. Evaluate the integral x2 ln x dx.


R

Solution. Use integration by parts, let

u = ln x, dv = x2 dx

1 x3
du = dx, v = .
x 3
Then,
Z Z
x ln x dx =
2
u dv
Z
= uv − v du
1 3 1 3 1
  Z
= x ln x − x · dx
3 3 x
1 3 1
Z
= x ln x − x2 dx
3 3
1 1
= x3 ln x − x3 + C.
3 9

2°. Suppose thatR f (1) = 3, f (5) = 13, f ′ (1) = −2, f ′ (5) = 4 and f ′′ is continuous. Find
the value of 15 xf ′′ (x) dx.
Solution. We use integration by parts,

u = x, dv = f ′′ (x) dx

du = dx, v = f ′ (x).
44 TECHNIQUES OF INTEGRATION

Then
Z 5 Z 5
xf ′′ (x) dx = u dv
1 1
Z 5
= uv|51 − v du
1
Z 5
′ 5
= xf (x) f ′ (x) dx

1 −
1
= 5f ′ (5) − 1f ′ (1) − f (x)|51
 

= [5(4) − 1(−2)] − [f (5) − f (1)] = 12.

R1
3°. Evaluate the integral 0 (1 + x2 )e−x dx.
Solution.
Z 1 Z 1
(1 + x2 )e−x dx = e−x + x2 e−x dx
0 0
Z 1 Z 1 Z 1
1
= e−x dx + x2 e−x dx = −e−x 0+ x2 e−x dx

0 0 0
1
Z 1 Z 1
−1 2 −x
= −e +e + 0
x e dx = 1 − + x2 e−x dx
0 e 0

R 1 2 −x
Using integration by parts for 0 x e dx

u = x2 , dv = e−x dx

du = 2x dx, v = −e−x

Z Z 1
x2 e−x dx = −x2 e−x − (−2x)e−x dx
0
1
1
Z 1 Z 1
= (−x2 e−x ) + 2 xe−x dx = − + 2 xe−x dx
0 0 e 0

Using integration by parts again for xe−x dx


R

u = x, dv = e−x dx

du = dx, v = −e−x
Then,

1 2
Z 1 1
Z 1 1
−x −x
xe dx = (−xe ) − (−e−x ) dx − − e−x =1− .
0 0 0 e 0 e

Now substituting back

1 1 2 6
Z 1   
(1 + x2 )e−x dx = 1 − + − +2 1− =3− .
0 e e e e
5.4 Solved Problems 45

4°. Use the Comparison Test to determine whether the integral is convergent or diverge.

3 + e−x
Z ∞
.
1 x

Solution. We have
3 3 + e−x
0≤ ≤ .
x x
R ∞ 3+e−x
Since 1∞
R∞ 1
3
dx = 3 1 x dx diverges, by the Comparison Test, 1 dx also
R
x x
diverges.
R ∞ 3+e−x
Thus, the integral 1 x diverges.
R ∞ e−√x
5°. Evaluate the integral 1

x
dx or say that it diverges.
Solution.
Z ∞ − √x Z b −√x
e e
√ dx = lim √ dx
1 x b→∞ 1 x
h √ ib
= lim −2e− x
[Integration by substitution]
b→∞ 1
 √ 
= lim −2e− b
+ 2e−1
b→∞

= −2 lim e− b
+ 2e−1 .
b→∞


Since e− b approaches 0 as b approaches infinity, the integral converges. Thus, we
have

Z ∞ −√x
e
√ dx = 2e−1 .
1 x
R5
6°. Evaluate the integral 1

3
2
5−x
dx or say that it diverges.
Solution.

2 2
Z 5 Z a
√ dx = lim √ .
1
3
5−x a→5− 1
3
5−x

Ra
Now, let’s compute 1

3
2
5−x
using integration by substitution

u = 5 − x, du = −dx

When
x = 1, u=4

and when
x = a, u = 5 − a.
46 TECHNIQUES OF INTEGRATION

Then,
2
Z a Z 5−a
du
√ = −2 √
1
3
5−x 4
3
u
Z 5−a
= −2 u−1/3 du
4
5−a
= −3u 2/3

q 4 √ 
3
= −3 3
(5 − a)2 − 4 .
2

Thus,
Z a
2
 q √
3

lim √ = lim −3 3 (5 − a)2 − 42
a→5− 1
3
5 − x a→5−

= 6 2.
3

R2 2
7°. Evaluate the integral 0 u ln u du or say that it diverges.
Solution.
" #2 
u3 1 
Z 2 Z 2 3
u
u2 ln u du = lim  ln u − · du [Integration by partsu = ln x, dv = x3 dx ]
0 a→0+ 3 a a 3 u

8 1
" # !
a3
Z 2
= lim ln 2 − ln a − u2 du
a→0+ 3 3 3 a
 #2 
8 1 u3
"
= lim  ln 2 −
3 3 3

a→0+
a
8 8
= ln 2 − .
3 9
8 8
Hence, the integral convergs to ln 2 − .
3 9
8°. Given the table of values of a function f . Use Trapezoidal rule to estimate the
integral 18 f (x) dx using these subintervals.
R

x 1 2 3 5 8
f (x) 2 3 4 3 1

Solution.
∆x1 = 2 − 1 = 1, ∆x2 = 3 − 2 = 1, ∆x3 = 5 − 3 = 2, ∆x4 = 8 − 5 = 3

1 1 5
Area1 = (f (1) + f (2)) · ∆x1 = (2 + 3) · 1 =
2 2 2
1 1 7
Area2 = (f (2) + f (3)) · ∆x2 = (3 + 4) · 1 =
2 2 2
1 1
Area3 = (f (3) + f (5)) · ∆x3 = (4 + 3) · 2 = 7
2 2
1 1
Area4 = (f (5) + f (8)) · ∆x4 = (3 + 1) · 3 = 6.
2 2

5 7
Approximation = Area1 + Area2 + Area3 + Area4 = + + 7 + 6 = 12.5.
2 2
5.5 Supplementary Problems 47

R2 √
9°. Use Simpson’s rule with n = 6 to approximate the integral 0 cos( x).
Solution.
b−a 2−0 1
∆x = = = ,
n 6 3


f (x0 ) = cos( 0) = cos(0) = 1,
q
f (x1 ) = cos( 1/3),
q
f (x2 ) = cos( 2/3),

f (x3 ) = cos( 1) = cos(1),
q
f (x4 ) = cos( 4/3),
q
f (x5 ) = cos( 5/3),

f (x6 ) = cos( 2),

√ ∆x h
Z 2 i
cos( x) dx ≈ 1 + 4f (x1 ) + 2f (x2 ) + 4f (x3 ) + 2f (x4 ) + 4f (x5 ) + f (x6 )
0 3
1h q q q
≈ 1 + 4 cos( 1/3) + 2 cos( 2/3) + 4 cos(1) + 2 cos( 4/3)
9 q √ i
+ 4 cos( 5/3) + cos( 2) .

5.5 Supplementary Problems

1°. Evaluate the integrals


(a) xe−3x dx.
R

(b) (x − 1) cos(πx) dx.



(c) ln( 3 x) dx.
R

R7
2°. Suppose that f (2) = 5, f (7) = 11, 2 f (x) dx = 8 and f ′ is continuous. Find
R7
2 xf (x) dx.

3°. Determine whether the integral is convergent or divergent.


R ∞ x+3
(a) 1

x4 −x
dx
R∞ 1
(b) 2 x ln x dx

4°. Evaluate the integral or say that it diverges.


R∞ 1
(a) 1 x1.2 dx.
R4 3
(b) 2

2x−4
dx.

5°. Given the table of values of a function f .

x 1 2 3 4 5
f (x) 2 3 1 3 2
48 TECHNIQUES OF INTEGRATION

R5
Estimate the integral 1 f (x) dx using:
(a) Trapezoidal rule.
(b) Simpson’s rule.

5.6 Multiple Choice Questions

1. What is the integration by parts formula?

(A) u dv = uv − v du
R R

(B) u dv =
R R
v du
(C) u dv = uv + v du
R R

(D) uv dx = u dx + v dx
R R R

2. Which of theR following functions would be appropriate to choose as u in integration


by parts for xex dx?

(A) x
(B) ex
(C) x + ex
(D) ln x

3. Which of the following improper integrals is convergent?


R∞ 1
(A) 1 x dx
R ∞ −x
(B) 0 e dx
(C) 1∞ √1x dx
R

R ∞ x2
(D) 1 5x+1 dx

4. Which of theR following functions would be appropriate to choose as u in integration


by parts for x ln x dx?

(A) ln x
(B) x
(C) x2
(D) 1
x
R4
5. Suppose that f (1) = 2, f (4) = 7 and f ′ is continuous. If 1 xf ′ (x) dx = 15, find the
value of 14 f (x) dx.
R

(A) 11
(B) 13
(C) 9
(D) 10
5.6 Multiple Choice Questions 49

6. Suppose thatR f (1) = 2, f (4) = 7, f ′ (1) = 5, f ′ (4) = 3 and f ′′ is continuous. Find


the value of 14 xf ′′ (x) dx.

(A) 2
(B) 3
(C) 4
(D) 5

7. Given the table of values of a function f . Use Trapezoidal rule to approximate the
integral 03 f (x) dx using these subintervals.
R

x 0 2 3
f (x) 2 3 4

(A) 7
(B) 8.5
(C) 9.5
(D) 8

8. Given the
R4
table of values of a function f . Use Simpson’s rule to approximate the
integral 0 f (x) dx using these subintervals.

x 0 1 2 3 4
f (x) 1 0 2 1 3

(A) 10
3
(B) 16
3
(C) 12
(D) 4
R ∞ −0.2x
9. Compute the improper integral 0 e dx or say that it diverges.

(A) 0.2
(B) 2
(C) 5
(D) The integral diverges

10. Calculate the improper integral or say that it is divergent.


x−1
Z 1

0 x

(A) − 13
(B) − 23
(C) − 43
(D) The integral is divergent

ANSWER: 1A 2A 3B 4A 5A 6A 7B 8D 9C 10C
50 TECHNIQUES OF INTEGRATION
Chapter 6

SYSTEMS OF LINEAR EQUATIONS

6.1 Solutions and Elementary Operations

• An equation of the form

c1 x1 + c2 x2 + · · · + cn xn = b (6.1)

is called a linear equation with n variables (or unknowns) x1 , x2 , · · · , xn and coeffi-


cients c1 , c2 , ..., cn and b are real numbers.

• A n-tuple of real numbers (s1 , s2 , ..., sn ) is called a solution to the (6.1) if

c1 s1 + c2 s2 + · · · + cn sn = b

• A finite collection of linear equations in the variables x1 , x2 , ..., xn is called a system


of linear equations in these variables.

• (s1 , s2 , ..., sn ) is called a solution to a system of equations if it is a solution to every


equation of the system.

6.2 Gaussian Elimination

• To solve a system of linear equations using Gaussian elimination, we start with the
augmented matrix representing the system. Let’s consider a system of m equations
with n variables:

a11 x1 + a12 x2 + · · · + a1n xn = b1






a21 x1 + a22 x2 + · · · + a2n xn = b2


..



 .
am1 x1 + am2 x2 + · · · + amn xn = bm

The augmented matrix for this system is


 
a11 a12 ··· a1n b1

 a21 a22 ··· a2n b2 

 .. .. .. .. .. 

 . . . . .


am1 am2 · · · amn bm

We perform row operations to transform this matrix into row-echelon form, where
each leading entry is 1 and is the only nonzero entry in its column. This process
involves three types of row operations:
1. Multiply a row by a nonzero constant.

51
52 SYSTEMS OF LINEAR EQUATIONS

2. Add a multiple of one row to another row.


3. Interchange two rows.
The goal is to introduce zeros below each leading one. Once the matrix is in (reduced)
row-echelon form, we can solve the system of equations through back substitution.
Consider the system of equations

2x − y = 3
(

x + 2y = 4
The augmented matrix of the system is
" #
2 −1 3
1 2 4

We can solve this system using a sequence of row elementary operations on the
augmented matrix
" # " # " #
2 −1 3 r1 ↔r1 1 2 4 −2r1 +r2 1 2 4
−−−−→ −−−−−→
1 2 4 2 −1 3 0 −5 −5
" # " #
− 15 ·r1 1 2 4 −2r2 +r1 1 0 2
−−−−→ −−−−−→
0 1 1 0 1 1

This matrix corresponds to the system of equations

x=2
(

y=1

So, (2, 1) is the unique the solution of the equation.

6.3 Homogeneous Equations

• A system of linear equations is said to be homogeneous if all the constant terms are
zero.
For example, a system of linear equations is homogeneous if it can be represented
in the form:
a11 x1 + a12 x2 + · · · + a1n xn = 0




a21 x1 + a22 x2 + · · · + a2n xn = 0


.
..



am1 x1 + am2 x2 + · · · + amn xn = 0

where aij are constants and x1 , x2 , . . . , xn are variables.

• If a homogeneous system of linear equations has more variables than equations, then
it has infinitely many solutions. Moreover, if the system has n variables and the rank
of the coefficient matrix is r, then there are n − r parameters in the solution.
6.4 Solved Problems 53

6.4 Solved Problems

1°. Solve the system 


x + y − z =0


2x + 4y − z =0

5x + 11y − 2z

=0
Solution. To solve the given system of equations, we start by representing it in
augmented matrix form and then perform row operations

1 1 −1 0
 

2 4 −1 0 
 

5 11 −2 0

Performing row operations to get the system into row-echelon form

1 1 −1 0 1 1 −1 0
   

 0 2 1 0  −→  0 2 1 0 
   
0 6 3 0 0 0 0 0

1 1 −1 0 1 0 − 23 0
   

−→  0 1 21 0  −→  0 1 12 0 
   
0 0 0 0 0 0 0 0

Now, the system has infinitely many solutions involving one parameter

x = 2 t
 3

y = −2t 1

z = t ∈ R.

2°. Find a, b, and c so that the system



x + ay + cz = 1


bx + cy + 2z = 3

ax + y − bz = 1

has the solution x = 1, y = −1, z = 2.


Solution. Since x = 1, y = −1, z = 2 is a solution to the system, we have
 
1 − a + 2c = 1 −a + 2c = 0

 

b−c+4=3 or b − c = −1
 
a − 1 − 2b = 1
 a − 2b = 2

We carry the augmented matrix of the system to a row-echelon matrix

−1 0 2 0 1 0 −2 0 1 0 −2 −1
     

 0 1 −1 −1  →  0 1 −1 −1  →  0 1 −1 −1 
     
1 −2 0 2 0 −2 2 2 0 0 0 0
54 SYSTEMS OF LINEAR EQUATIONS

Hence, the solution of the system is



a = −1 + 2c


b = −1 + c

c ∈ R

3°. Find all values of of a such that the system



ay + z =0


x + (a − 1)y + z =0

x − ay + (a − 1)z

=0

has a unique solution.


Solution. To solve the given system of equations, we start by representing it in
augmented matrix form and then perform row operations

0 1 1 0
 

 1 a−1 1 0 
 
1 −a a − 1 0

Performing row operations to get the system into row-echelon form

0 1 0 1 a−1 1 0 1 a−1 1 0
     
a
 1 a−1 1 0  −→  0 a 1 0  −→  0 a 1 0 
     
1 −a a − 1 0 1 −1 a − 1 0 0 −a a − 2 0

We consider two cases:


1. a = 0. The system becomes

1 a−1 1 0 1 a−1 1 0
   

 0 0 1 0  −→  0 0 1 0 
   
0 0 −2 0 0 0 0 0

which has infinitely many solutions.


2. a ̸= 0. The system becomes

1 a−1 1 0
 

 0 a 1 0 
 
0 0 a−1 0

which has a unique solution if and only if a − 1 ̸= 0 or a ̸= 1.


In conclusion, the system has a unique solution when a ̸= 0 and a ̸= 1.

4°. Solve the system 


x + y − 2z + 2w

 =2
−2x + y − w = −3

3x − 3y + 2z + 2w

=4
6.4 Solved Problems 55

Solution.
To solve the given system of equations, we start by representing it in augmented
matrix form and then perform row operations

1 1 −2 2 2
 

 −2 1 0 −1 −3 
 
3 −3 2 2 4

Performing row operations to get the system into row-echelon form

1 1 −2 2 2 1 1 −2 2 2
   

−2 1 0 −1 −3 −→ 0 3 −4 3 1 
   
  
3 −3 2 2 4 0 −6 8 −4 −2

1 1 −2 2 2 1 1 −2 2 2
   

−→  0 1 −4/3 1 1/3  −→  0 1 −4/3 1 1/3 


   
0 0 0 2 0 0 0 0 1 0

1 0 −2/3 0 5/3
 

−→  0 1 −4/3 0 1/3  .
 
0 0 0 1 0

Now, the system has infinitely many solutions involving one parameter

x = 53 + 23 t



y = 1 + 4 t

3 3


z = t ∈ R
w = 0.


5°. Find the rank of the matrix


1 −2 3
 
 2 −1 2
 
−1 2 0
 

3 −2 5
Solution. Carry the matrix to a row-echelon matrix and then the number of non-
zero rows is the rank of the matrix.

1 −2 3 1 −2 3 1 −2 3
     
 2 −1 2 0 3 −4 0 4 −4
 −→   −→ 
     
−1 2 0 0 0 3 0 0 3
 

3 −2 5 0 4 −4 0 3 −4

1 −2 3 1 −2 3
   
0 1 −1 0 1 −1
−→   −→ 
   
0 0 3 0 0 1

0 0 −1 0 0 0

Hence, the rank of the matrix is 3.

6°. Solution.
56 SYSTEMS OF LINEAR EQUATIONS

6.5 Supplementary Problems

1°. Solve the system


x+y−z+w =0
(

2x + 2y − z =0

2°. Find a, b, and c so that the system



x + ay + cz = −1


ax + cy + 2z = −1

ax + y − bz = −4

has the solution x = 1, y = 0, z = 2.



x + y + az =2


3°. Find all values of of m such that the system 2x + y + az =3 has no solu-

x + (a2 − 25)z

=a+6
tion.
2 2 1
 

4°. Solve the system (A − 3I)x = 0, where I is the identity matrix and A = 0 1 0 .
 
1 0 2

1 −2 3 1
 

5°. Find the rank of the matrix  3 −5 7 4 .


 
−1 1 −1 −2

6.6 Multiple Choice Questions

1. Find the
" solution to#the system of linear equations represented by the augmented
2 −1 2
matrix .
1 −1 3

(A) (1, 4)
(B) (−1, −4)
(C) (−4, −1)
(D) No solution

2. Which of the following is a valid elementary row operation?

(A) Adding a multiple of one row to another row


(B) Multiplying a row by a nonzero constant
(C) Inter change two rows
(D) All of the above

3. Which of the following matrices is in row-echelon form?


6.6 Multiple Choice Questions 57

1 2 3
 

(A) 0 1 4
 
0 0 5
1 2 3
 

(B) 0 0 0
 
0 1 0
1 2 3
 

(C) 1 0 0
 
0 0 1
1 0 1
 

(D) 0 1 3
 
0 0 0

1 2 −1
 

4. Find the rank of the matrix 2 3 4 .


 
1 1 1

(A) 2
(B) 2
(C) 3
(D) Cannot be determined

5. Suppose the rank of the augmented matrix of a system of homogeneous linear equa-
tions is known to be 7. How many parameters are there in the solution of the system
if the system consists of 13 equations and 9 variables?

(A) 2
(B) 3
(C) 4
(D) 6

6. Find all values of α such that the system



x − z

 =1
x+y+z =0

2x + y + (α2 − 1)z

=α+2

has infinitely many solutions.

(A) α = 1
(B) α = −1
(C) α = 2
(D) α = −2

7. Solve the system


=1
(
x−z
x+y+z =2
58 SYSTEMS OF LINEAR EQUATIONS

(A) (x, y, z) = (1 − t, 1 − 2t, t), t ∈ R


(B) (x, y, z) = (1 + t, 1 − 2t, t), t ∈ R
(C) (x, y, z) = (1 + t, 2 + 2t, t), t ∈ R
(D) (x, y, z) = (1 − t, 1 + 2t, t), t ∈ R

8. Find α such that the rank of the matrix is 2.

1 2 −1
 

2 1 α 
 
0 1 3

(A) 7
(B) −3
(C) −11
(D) No such α

9. How many parameters are there in the general solution of the system

a + b − c + d − e =0


2a + b − 3c + e =0

a + 2b + 3d − 4e

=0

(A) 5
(B) 4
(C) 3
(D) 2

ANSWER: 1B 2D 3D 4A 5A 6B 7B 8C 9C
Chapter 7

MATRIX ALGEBRA
7.1 Matrix Addition, Scalar Multiplication, and Matrix Mul-
tiplication

• Matrix Addition
Matrix addition is performed by adding corresponding elements of two matrices of
the same size. Let A and B be two m × n matrices. The sum of A and B, denoted
by A + B, is calculated as

   
a11 a12 ··· a1n b11 b12 ··· b1n
 a21

a22 ··· a2n 
  b21

b22 ··· b2n 
A+B = + .

 .. .. .. ..  .. .. .. 
.   ..

 . . . . . . 

am1 am2 · · · amn bm1 bm2 · · · bmn

a11 + b11 a12 + b12 a1n + b1n


 
···
 a21 + b21 a22 + b22 ··· a2n + b2n
 
=

.. .. .. .. 

 . . . .


am1 + bm1 am2 + bm2 · · · amn + bmn

• Scalar Multiplication
Let k be a scalar and A be an m × n matrix. The scalar product of k and A, denoted
by kA, is calculated as

   
a11 a12 ··· a1n ka11 ka12 ··· ka1n
 a21

a22 ··· a2n 
  ka21

ka22 ··· ka2n 
kA = k ·  = .

 .. .. .. ..  .. .. .. 
.   ..

 . . . . . . 

am1 am2 · · · amn kam1 kam2 · · · kamn

• Matrix Multiplication
Let A be an m × p matrix and B be a p × n matrix. The product of A and B,
denoted by AB, is calculated as

    
a11 a12 ··· a1p b11 b12 · · · b1n c11 c12 · · · c1n
 a21

a22 ··· a2p  b21 b22 · · · b2n   c21 c22 · · · c2n 
  
AB =   =  ..

 .. .. .. .. 
  .. .. . . ..  .. .. .. 
 
 . . . .  . . . .   . . . . 

am1 am2 · · · amp bp1 bp2 · · · bpn cm1 cm2 · · · cmn

Pp
where each element cij is computed as cij = k=1 aik bkj .

59
60 MATRIX ALGEBRA

• Matrix Transposition
The transpose of a matrix A = [aij ] of size m × n is matrix B = [bij ] of size n × m
and determined by
bij = aji

for i = 1, . . . , n and j = 1, . . . , m.

7.2 Matrix Inverses

• The inverse of an n × n matrix A, denoted by A−1 , is an n × n matrix that, when


multiplied by A, yields the identity matrix In . In other words, we have

A · A−1 = A−1 · A = In (7.1)

• Not all matrices have inverses. A matrix is said to be invertible if it has an inverse.

• For a 2 × 2 matrix A,

" #
a b
A=
c d

The inverse of A, denoted by A−1 , is given by

1
" #
−1 d −b
A =
ad − bc −c a

provided ad − bc ̸= 0. Otherwise, A does not have an inverse.

• Matrix Inversion Algorithm


Given a square matrix A of size n×n, we can find its inverse A−1 using the following
steps:

1. Form the augmented matrix [A | I], where I is the n × n identity matrix.


2. Apply Gaussian elimination to the augmented matrix [A | I] to transform the
left part (matrix A) into the identity matrix I. Perform the same row operations
on the right part (matrix I).
3. Once the left part is transformed into I, the right part will be A−1 .
4. If A is invertible, then A−1 will be obtained; otherwise, if A is not invertible,
the algorithm will fail to find the inverse.

Remarkable Properties on Matrices


7.3 Linear Transformations 61

A + (B + C) = (A + B) + C
A + (B + C) = (A + B) + C
(cA)B = c(AB) = A(cB)
A0 = 0A = 0
AI = IA = A
A(B + C) = AB + AC
(A + B)T = AT + B T
(AB)T = B T AT
(AB)−1 = B −1 A−1

7.3 Linear Transformations

• Linear Transformation
A transformation T from Rn to Rm is called linear transformation if the following
properties hold for any vectors v1 , v2 ∈ Rn and any scalar k:

1. T (v1 + v2 ) = T (v1 ) + T (v2 )


2. T (kv1 ) = kT (v1 )

• Representation using Matrix


We can represent the linear transformation T using a matrix A, where each column
of A corresponds to the image of the standard basis vectors of R2 under T . For
example, let’s consider a linear transformation T from R2 to R2 . If T is defined by
" #! " #
x 2x + y
T = ,
y x−y
then the matrix of the transformation is
" #
2 1
A= .
1 −1

Note that T (v) = Av for v ∈ R2 .


The composition of T and S is defined by

(T ◦ S)v = T (S(v)).

The matrix of the composition of two transformations S and T is

M := MS · MT ,

where MS , MT are the matrix of the linear transformations S and T, respectively.


The matrix of the inverse transformation of an invertible linear tranformation T −1
of T is A−1 , where A is the matrix of T .
62 MATRIX ALGEBRA

7.4 Solved Problems


−1 2
 
" #
1 2 −1
1°. Let A = , B =  3 2 .
 
−1 3 0
−2 5
Compute AB and BA.
Solution.

# −1 2
 
" " #
1 2 −1  7 1
AB =  3 2 = ,

−1 3 0 10 4
−2 5

−1 2 " −3 4 1
   
#
 1 2 −1
BA =  3 2 =  1 12 −3 .
  
−1 3 0
−2 5 −7 11 2

2°. Find the "inverse


#
of each of these matrices.
3 2
(a) M =
1 4
1 0 1
 

(b) N = 1 −1 0
 
0 2 1
Solution.
(a) The inverse of M , denoted as M −1 , is
1
" #
4 −2
M −1 =
3·4−1·2 −1 3

1 4 −2
" #
−1
M = .
10 −1 3

(b) To find the inverse of N , we use the inversion algorithm

1 0 1 1 0 0 1 0 1 1 0 0 1 0 1 1 0 0
     

1 −1 0 0 1 0 → 0 −1 −1 −1 1 0 → 0 1 1 1 −1 0 
     
    
0 2 1 0 0 1 0 2 1 0 0 1 0 2 1 0 0 1

1 0 1 1 0 0 1 0 1 1 0 0 1 0 1 1 0 0
     

→ 0 1 1 1 −1 0  →  0 1 1 1 −1 0  →  0 1 1 1 −1 0 
     
0 0 −1 −2 2 1 0 0 1 2 −2 −1 0 0 1 2 −2 −1

1 0 0 −1 2 1
 

→  0 1 0 −1 1 1 
 
0 0 1 2 −2 −1

−1 2 1
 

Hence, the inverse of N is N −1 = −1 1 1 .


 
2 −2 −1
7.4 Solved Problems 63

3°. Find a matrix A such that


" #
−1 4 −1
(A + 5I)
T
=
2 0

Solution.
" #
−1 4 −1
(A + 5I)
T
=
2 0
" #−1 " #
4 −1 0 12
(A + 5I) =
T
=
2 0 −1 2
" # " # " #
0 21 1 0 −3 12
AT = −5 = .
−1 2 0 1 −1 −1
" # " #
−3 −1 −3 −1
Hence, 1 is the inverse of A or A = 1 .
2 −1 2 −1

4°. Suppose A is a square matrix such that A2 − 3A + 2I = 0. Find A−1 in terms of A.


Solution. Notice that a square matrix B is an inverse of A if

AB = BA = I

. We have the following

A2 − 3A + 2I = 0
A(A − 3I) = −2I
1 3
A(− A + I) = I.
2 2
Hence, A−1 = − 12 A + 32 I.
5°. Give an example of two symmetric matrices whose product is not symmetric.
Solution. Recall that a square matrix M is symmetric if M T = M . Consider two
symmetric matrices " # " #
1 3 0 1
A= , B=
3 0 1 2
Then, the product is " #" # " #
1 3 0 1 3 7
AB = = ,
3 0 1 2 0 3

which is NOT symmetric.


6°. Let A be a square matrix. Prove that if A2 is invertible, then so is A.
Solution. Since A2 is invertible, there exists a square matrix B such that

A2 B = BA2 = I,

where I is the identity matrix of the same size as A.


Let C := AB, we have
A2 B = AAB = AC = I,
which means A is invertible and C is the inverse of A.
64 MATRIX ALGEBRA

" #2 " #
a b 1 0
7°. (a) Find a, b such that = .
0 1 0 1
" #2025
−1 2
(b) Compute .
0 1
Solution.
(a) We have
" #2 " #" # " #
a b a b a b a2 ab + b
= =
0 1 0 1 0 1 0 1
Therefore,
" #2 " #
a b 1 0
=
0 1 0 1
if and only if a2 = 1 and ab + b = 0. Solving these conditions, we obtain
a = 1, b = 0
or
a = −1, b is arbitrary number.
(b) From the part (a), when a = −1 and b = 2, we have
" # " #
−1 2 1 0
A =
2
= =I
0 1 0 1

Hence, A2025 = |A2 ·{z


· · A2} ·A = I 1012 A = A.
1012 times

8°. Consider the linear transformation T : R2 → R2 defined by T (x) = Ax where


" #
1 2
A=
−1 3
" #
1
(a) Find the image of the vector v = under the transformation T .
1
(b) Find the pre-image of (u) = [3 2]T under the transformation T .
Solution.
(a) The image of v under T is given by T (v) = Av
" #" # " #
1 2 1 3
T (v) = =
−1 3 1 2

(b) We want to find a vector w such that Tw = u.


or equivalently, Aw = u.
To find w, we multiply both sides by A−1
Aw = u
−1
A Aw = A−1 u
w = A−1 u
1 3 −2
" #" # " #
3 1
= = .
5 1 1 2 1
7.4 Solved Problems 65

Note that (b) can be solved as follow.


" # " # " #
x x x + 2y
We have T =A = .
y y −x + 3y
So, T (a, b) = (a + 2b, −a + 3b) = (3, 2) if and only if

a + 2b =3 =1
( (
a

−a + 3b = 2 b =1

9°. Given two linear transformations

T : R3 → R2 , T (x, y, z) = (x − 2y, y + z)

S : R2 → R2 , T (x, y) = (x + y, 2x − y)

Find the composition transformations S ◦ T and S −1 ◦ T.


Solution. (a) In the standard bases of R2 and R3 , the matrices of T, S, respectively,
are " # " #
1 −2 0 1 1
MT := , MS :=
0 1 1 2 −1

Then, the matrix of S ◦ T is


" #" # " #
1 1 1 −2 0 1 −1 1
MS · MT = = .
2 −1 0 1 1 2 −5 −1

Therefore,
   
x " # x " #
1 −1 1   x−y+z
(S ◦ T ) y  = y  =
 
2 −5 −1 2x − 5y − z
z z

or
(S ◦ T )(x, y, z) = (x − y + z, 2x − 5y − z).

Next, the matrix of S −1 is


" #
1 1
MS−1 = 3
2
3
3 − 13

and then S −1 ◦ T has the matrix

1 1 −1 1
" #" # " #
1 1
1 −2 0
3 3 = .
2
3 − 13 0 1 1 3 2 −5 −1

Hence, (S −1 ◦ T )(x, y, x) = 31 (x − y + z, 2x − 5y − z).


66 MATRIX ALGEBRA

7.5 Supplementary Problems


" #
1 3 0
1°. Let A = .
2 −1 2
Compute AAT and AT A.

2°. Find the inverse of each of these matrices.


" #
1 5
(a) M = .
−1 3
1 0 1
 

(b) N = 0 −1 1 .
 
1 1 1
3°. Find a matrix A such that
" #
−1 2 1
(A − 3I)
T
= .
4 3

4°. Suppose A is a square matrix such that −A2 + A + 3I = 0. Find A−1 in terms of A.

5°. Let A, B be square matrices. Prove that if AB is invertible, then so is A.


" #2025
1 3
6°. Compute .
0 −1

7°. Consider the linear transformation T : R2 → R2 defined by T (x) = Ax where


" #
1 1
A= .
2 3
" #
1
(a) Find the image of the vector v = under the transformation T .
2
(b) Find the pre-image of (u) = [1 1]T under the transformation T .

8°. Given two linear transformations

T : R2 → R2 , T (x, y) = (x − 2y, y)

S : R2 → R2 , T (x, y) = (x + y, 2x − y).

Find the composition transformations S ◦ T, S ◦ S and T ◦ S.

7.6 Multiple Choice Questions


1. Given the matrices with their sizes,

P3×4 , Q4×5 , R3×6 .

Which of the following matrices exist?


7.6 Multiple Choice Questions 67

(A) P QR
(B) P RQ
(C) P T RQ
(D) RT P Q

2. Find the (2, 3)−entry of the product


# 3 1 5 2
 
"
1 2 −1 
4 −1 3 −2 .

0 1 3
1 3 2 5

(A) 5
(B) 7
(C) 9
(D) 11
" # " #
1 2 a b
3. Multiply A = and B = and then add up the elements on the main
1 −1 b a
diagonal. What is the result?

(A) a + 2b
(B) 3b
(C) 2a + b
(D) 3b − a
" #
1 0
4. Given V = .
0 −1
Compute V 2025 .

(A) V T
(B) −V
(C) V
(D) I

1 1 2
 

5. Given A = a 3 −1 .
 
0 4 5
For what value(s) of a will a row interchange be required during Gaussian elimina-
tion?

(A) 0
(B) 1
(C) 2
(D) 3

6. Let A, B, and C denote invertible matrices of size n × n. Which of the following


statement(s) is (are) always TRUE?
68 MATRIX ALGEBRA

(A) (A + B)(A − B) = A2 − B 2
(B) AB = AC implies B = C
(C) AB = 0 implies A = 0 or B = 0
(D) (ABC)−1 = C −1 B −1 A−1
" #
2 1 −1
7. Find AAT where A = .
1 −1 0
" #
6 4
(A)
4 6
" #
4 6
(B)
6 2
" #
6 4
(C)
4 2
" #
2 6
(D)
6 4

8. Which of the following matrices is NOT invertible?


" #
0 1
(A)
1 0
" #
1 2
(B)
2 1
" #
1 2
(C)
3 4
" #
1 1
(D)
2 2

9. Suppose A is a square matrix such that A2 − 3A + 2I = 0. Find the inverse of A in


terms of A, where I is the indentity matrix.

(A) A−1 = 21 (I − 3A)


(B) A−1 = 12 (3A − I)
(C) A−1 = A − 3I
(D) A−1 = 3I − A

1 0 0 1
 
0 2 0 0
10. Find the sum of all entries on the main diagonal of the inverse of A =  .
 
0 0 5 3
0 0 5 1

(A) 6
5
(B) 3
5
(C) 2
5
(D) 1
5
7.6 Multiple Choice Questions 69

11. Let T : R3 → R2 be a linear transformation. Suppose A is a matrix such that


T (v) = Av for v ∈ R3 . What is the size of A?

(A) 3 × 3
(B) 3 × 2
(C) 2 × 3
(D) 2 × 2

12. Given T (x, y) = (x − y, 2x + y). Suppose T (a, b) = (3, 3), what is the value of a + b?

(A) 4
(B) 3
(C) 2
(D) 1

ANSWER: 1A 2C 3B 4C 5D 6D 7C 8D 9A 10A 11C 12D


70 MATRIX ALGEBRA
Chapter 8

DETERMINANTS AND
DIAGONALIZATION
8.1 Determinants
• Determinants and Properties
The determinant of a square matrix A, denoted by det(A) or |A|, is a number that
can be computed from its elements. The determinant of a 2×2 matrix A is calculated
as
det(A) = |A| = ad − bc, (8.1)
" #
a b
where A = .
c d
For a 3 × 3 matrix A, the determinant can be computed using a cofactor expansion
as
a a a a a a
det(A) = |A| = a11 22 23 − a12 21 23 + a13 21 22 . (8.2)
a32 a33 a31 a33 a31 a32

The determinant has the following properties.

1. If A has a row or column of zeros, then det(A) = 0.


2. Interchanging two rows (or columns) of A changes the sign of det(A).
3. If A has two identical rows or columns, then det(A) = 0.
4. If A is triangular (either upper or lower), then det(A) is the product of the
entries on the main diagonal.
5. If B is obtained from A by multiplying a row or column of A by k, then
det(B) = k det(A).
6. If B is obtained from A by adding a multiple of one row (or column) to another
row (or column) then det(B) = det(A).
7. det(AT ) = det(A).
8. If A is invertible, then det(A−1 ) = det(A) .
1

9. det(AB) = det(A)det(B).
10. det(cA) = cn det(A), where A is an n × n matrix and c ∈ R.

• Adjugate Matrices
The (i, j)-cofactor corresponding to the (i, j)-entry of an n × n matrix A is defined
by
cij (A) = (−1)i+j det(Aij ), (8.3)
where Ai,j is the submatrix of size (n − 1) × (n − 1) obtained from the matrix A by
deleting the row i and column j of A.
Given an n × n matrix A. The adjugate matrix of A, denoted by adj(A) is the
transpose of the matrix of all (i, j)-cofactors of the matrix A. That is,

71
72 DETERMINANTS AND DIAGONALIZATION

adj(A) = [cij (A)]T , (8.4)

where cij (A) is the (i, j)-cofactor corresponding to the (i, j)-entry of the matrix A.
If A is an invertible matrix, then

1
A−1 = · adj(A). (8.5)
det(A)

In general, we have

A · adj(A) = det(A) · I, (8.6)


where I is the identity matrix.

8.2 Diagonalization and Eigenvalues

• Eigenvalues and Eigenvectors


Let A be a square matrix. An eigenvalue λ of A is a scalar that satisfies the equation

Av = λv (8.7)

where v is a nonzero vector called the eigenvector corresponding to λ.


The eigenvalues of A can be found by solving the characteristic equation

det(A − λI) = 0,
where I is the identity matrix of the same size as A.
Once the eigenvalues λ1 , λ2 , . . . , λn are found, the corresponding eigenvectors v1 , v2 , . . . , vn
can be obtained by solving the equation Avi = λi vi for each eigenvalue.

• Diagonalization
A square matrix A is said to be diagonalizable if it can be written in the form

A = P DP −1 , (8.8)

where P is a matrix whose columns are the eigenvectors of A, and D is a diagonal


matrix containing the corresponding eigenvalues.
If A has n linearly independent eigenvectors, then it can be diagonalized. This
implies that A has n distinct eigenvalues.
Example
Let’s consider a matrix A
" #
2 −1
A= .
−1 2

The eigenvalues of A can be found by solving the characteristic equation


8.3 Solved Problems 73

det(A − λI) = 0

2 − λ −1
= 0.
−1 2 − λ

Solving this equation yields the eigenvalues λ1 = 1 and λ2 = 3.


The corresponding eigenvectors can be found by solving the equation Avi = λi vi
for each eigenvalue. Then, we can form the matrix P using these eigenvectors,
and the diagonal matrix D containing the eigenvalues. Finally, we can obtain the
diagonalization of A as P DP −1 .

8.3 Solved Problems


1°. Find all value(s) of x such that the matrix

1 2 −1
 

A = x −2 1 
 
0 x 2

is invertible.
Solution. The matrix A is invertible if and only if det(A) ̸= 0.
We find the determinant of the matrix using Rule of Sarrus for a 3 × 3 matrix and
obtain
det(A) = −x2 − 5x − 4.

Hence, det(A) ̸= 0 when x ̸= −1 and x ̸= −4.


Therefore, A is invertible when x ̸= −1 and x ̸= −4.

2°. Evaluate the determinant of the matrix


1 2 −1 0
 
2 −1 1 3
A= .
 
0 5 2 4
3 1 7 1

Solution. We use row elementary operations as follow.

1 2 −1 0 1 2 −1 0 1 2 −1 0
2 −1 1 3 0 −5 3 3 0 −5 3 3
det(A) = = =
0 5 2 4 0 5 2 4 0 0 5 7
3 1 7 1 0 −5 10 1 0 0 −7 −2

To find the derteminant of the last matrix, we can use the cofactor expansion along
the first column and obtain
−5 3 3
5 7
det(A) = 1 · 0 5 7 = 1 · (−5) · = 1 · (−5) · [5(−2) − (−7)7] = −195.
−7 −2
0 −7 −2
74 DETERMINANTS AND DIAGONALIZATION

3°. Suppose that


a b c
x y z = 5.
p q r

Find
2x − 3a 4a p − x
2y − 3b 4b q − y
2z − 3c 4c r − z

Solution. We use elementary operations and properties of the determinant.

2x − 3a 4a p − x 2x − 3a a p − x 2x a p − x
2y − 3b 4b q − y = 4 2y − 3b b q − y = 4 2y b q − y
2z − 3c 4c r − z 2z − 3c c r − z 2z c r − z

x a p−x x a p a x p
= 4(2) y b q − y = 4(2) y b q = 4(2)(−1) b y q
z c r−z z c r c z r

a b c
= 4(2)(−1) x y z = 4(2)(−1)(5) = −40.
p q r

4°. Find the characteristic and the eigenvalues of the matrix

0 3 3
 

A = 3 0 3 .
 
3 3 0

Solution. To find the characteristic polynomial of the matrix A we need to compute


the determinant of A − λI.
We compute A − λI

0−λ 3 3 −λ 3 3
   

A − λI =  3 0−λ 3  =  3 −λ 3 
   
3 3 0−λ 3 3 −λ

Next, we find the determinant of A − λI

−λ 3 3
det(A − λI) = 3 −λ 3
3 3 −λ

and obtain

−λ 3 3 3 3 −λ
det(A − λI) = (−λ) × −3× +3×
3 −λ 3 −λ 3 3

= −λ3 + 27λ + 54.


8.3 Solved Problems 75

Therefore, the characteristic polynomial of A is −λ3 + 27λ + 54 = 0.


Next, to find the eigenvalues of A, we find all roots of the characteristic polynomial
of A.
This characteristic polynomial can be factored as −(λ + 3)(λ − 6)2 = 0.
So, the eigenvalues are λ1 = −3 and λ2 = 6 (with multiplicity 2).

5°. Find all eigenvectors corresponding to the eigenvalue −3 of the matrix

0 3 3
 

3 0 3 .
 
3 3 0

Solution. To find all eigenvectors corresponding to the eigenvalue −3 of the matrix


A, we need to solve the equation (A + 3I)x = 0.
Solving (A + 3I)x = 0 gives

3 3 3 x1 0
    

3 3 3 x2  = 0
    
3 3 3 x3 0

This system of equations reduces to the single equation x1 + x2 + x3 = 0.


The solutions to this equation form a one-dimensional subspace. Any non-zero vector
x in this subspace is an eigenvector corresponding to the eigenvalue −3.
So,
 the eigenvectors
 corresponding to the eigenvalue −3 are all vectors of the form
−a − b
 b , where 0 ̸= a, b ∈ R.
 
a
6°. Suppose a 2×2 matrix A has two eigenvalues 3, 1 and two corresponding eigenvectors
[1 1]T , [1 − 1]T . Find the matrix A.
Solution. Since A has size 2 × 2 and two distinct eigenvalues, A is diagonalizable
and there exists an invertible P whose columns are eigenvectors of A such that
A = P DP −1 , where D is a diagonal matrix consisting of eigenvalues of A.
" # " # " #
1 1 3 0 −1 −1
Let P = ,D = , then P −1 = − 21 and
1 −1 0 1 −1 1

1 1 1
" #" #" # " #
−1 3 0 −1 −1 2 1
A = P DP =− = .
2 1 1 0 1 −1 1 1 2

We can find the matrix A as follow.


" #
a b
Given that A = has two eigenvalues 3 and 1, and their corresponding eigen-
c d
" # " #
1 1
vectors are and respectively, we can write the eigenequations as
1 −1
" # " # " # " #
1 1 1 1
A =3 , A =1
1 1 −1 −1

Expanding these equations gives


76 DETERMINANTS AND DIAGONALIZATION

" #" # " #


a b 1 3
=
c d 1 3
" #" # " #
a b 1 1
=
c d −1 −1

This gives us the following system of equations

a + b = 3, c + d = 3, a − b = 1, c − d = −1.

Solving this system of equations gives us the elements of matrix A

a = 2, b = 1, c = 1, d = 2.

So, the matrix A is


" #
2 1
A= .
1 2

7°. Find all eigenvalues of the matrix AT A if


" #
1 0 1
A= .
0 1 1

Solution. First, we calculate S := AT A

1 0 " 1 0 1
   
#
 1 0 1
S = 0 1 = 0 1 1
  
0 1 1
1 1 1 1 2

Next, the characteristic polynomial of S is

1−x 0 1
cS (x) = det(xI − A) = 0 1−x 1 = x(1 − x)(x − 3).
1 1 2−x

Next,to find the eigenvalues of S, we find the roots of cA (x). Then, we obtain the
eigenvalues of S which are x1 = 0, x2 = 1, and x3 = 3.

8.4 Supplementary Problems

1°. Find all value(s) of x such that the matrix

0 x −4
 

A = 3 7 −1
 
1 4 1

is NOT invertible.
8.5 Multiple Choice Questions 77

2°. Find the adjugate matrix of the matrix

1 0 2
 

2 −1 1 .
 
3 1 1

3°. Evaluate the determinant of the matrix


1 4 −2 −2
 
0 5 −1 −3
A= .
 
3 −4 1 4
5 3 0 1

4°. Suppose that


a b c
x y z = 3.
p q r

Find
−2x + p 4a x − 3a
−2y + q 4b y − 3b
−2z + r 4c z − 3c

5°. Find the characteristic and the eigenvalues of the matrix

2 2 1
 

A = 0 1 0 .
 
1 0 2

6°. Find all eigenvectors corresponding to the eigenvalue 1 of the matrix


 
−7 −8 −8
 4 5 4 .
 
2 2 3

7°. Suppose a 2×2 matrix A has two eigenvalues 2, 1 and two corresponding eigenvectors
[2 1]T , [1 − 1]T . Find the matrix A.

8°. Find all eigenvalues of the matrix AAT if


" #
1 0 1
A= .
0 1 1

8.5 Multiple Choice Questions

1. Which of the following matrices is invertible?

(A) A matrix with determinant 0


(B) A square matrix with all non-zero entries
(C) A square matrix with non-zero determinant
78 DETERMINANTS AND DIAGONALIZATION

(D) A matrix with eigenvalue 0


" #" #" #
1 3 2 1 1 3
2. What is the determinant of the matrix ?
2 5 1 2 2 5

(A) 1
(B) 2
(C) −3
(D) 3

3. Which of the following is a property of eigenvectors?

(A) They are unique for each eigenvalue


(B) They are always orthogonal to each other
(C) They can be scaled by any non-zero scalar
(D) They are always linearly independent

1 2 3
 

4. Find the determinant of the matrix 0 2 4


 
1 0 1

(A) 2
(B) 1
(C) 3
(D) 4

1 2 1
 

5. Find the (2, 3)-cofactor of the matrix −1 2 5


 
1 0 4

(A) 2
(B) 1
(C) 3
(D) 4

3 1 1
 

6. Find the (2, 3)-entry of A−1 if A = −1 2 1


 
1 1 1

(A) 2
(B) 4
(C) −2
(D) −4

7. Suppose A, B are 3 × 3 matrices such that det(A−1 ) = 2, det(B) = 5. Find


det(2A2 B T ).
8.5 Multiple Choice Questions 79

(A) 5
2
(B) 20
(C) 5
(D) 10

8. Suppose that
a b c
x y z = 7.
p q r

Find
2x − a 3a x − 5p
2y − b 3b y − 5q
2z − c 3c z − 5r

(A) 42
(B) 210
(C) −42
(D) −210
" #
2 1
9. What is the characteristic polynomial of the matrix ?
1 2

(A) x2 − 4x + 3
(B) x2 − 3x + 2
(C) x2 − 4x − 3
(D) x2 − 3x − 2
" #
3 2
10. What is the characteristic equation of the matrix ?
1 4

(A) x2 − 7x − 10 = 0
(B) x2 − 7x + 10 = 0
(C) x2 + 7x + 10 = 0
(D) x2 + 7x − 10 = 0

11. What are the eigenvalues of the matrix

1 0 3
 

1 1 5 ?
 
1 0 −1

(A) 2, 1, −2
(B) 1, −1
(C) 3, 2, 1
(D) 2, 1, −1
80 DETERMINANTS AND DIAGONALIZATION

12. Which of the following vectors is an eigenvector of the matrix

1 0 3
 

1 1 5 ?
 
1 0 −1

(A) [1 2 − 1]T
(B) [3 8 1]T
(C) [5 −1 2]T
(D) [1 0 1]T

ANSWER: 1C 2D 3C 4D 5A 6C 7D 8D 9A 10B 11A 12B


Chapter 9

THE VECTOR SPACE Rn


9.1 R3 and Cross Product
 
x1
• The dot product, also known as the scalar product, of two vectors v =  y1  and
 
z1
 
x2
w =  y2  in R3 is defined as
 
z2

v · w = x 1 x 2 + y1 y2 + z 1 z2

• The length of a vector v = [x y z]T in R3 is determined by


q
∥v∥ = x2 + y 2 + z 2 . (9.1)

• The angle between two vectors u, v is θ ∈ [0, π] determined by


u·v
cos θ = . (9.2)
∥u∥v∥∥

• The line with directional vector d = (a, b, c) through the point P (x0 , y0 , z0 ) is given
by      
x x0 at
y  =  y0  +  bt  (9.3)
     
z z0 ct
where t ∈ R.

• The plane with normal vector n = (a, b, c) through the point p0 = (x0 , y0 , z0 ) is
given by
a(x − x0 ) + b(y − y0 ) + c(z − z0 ) = 0 (9.4)
or in vector form, n · (p − p0 ) = 0, where t ∈ R.
 
x1
• The cross product, also known as the vector product, of two vectors v =  y1  and
 
z1
 
x2
w =  y2  in R3 is defined as
 
z2
 
y1 z2 − z1 y2
v × w = z1 x2 − x1 z2  (9.5)
 
x1 y2 − y1 x2

• The cross product of two vectors results in a vector that is orthogonal (perpendicular)
to both v and w.

81
82 THE VECTOR SPACE Rn

   
x1 x2
• The cross product of two vectors v =  y1  and w =  y2  in R3 can be expressed
   
z1 z2
as a determinant

i x1 x2
v × w = j y1 y2 (9.6)
k z2 z2

where i, j, and k are the standard basis vectors in R3 .


Expanding the determinant, we get

v × w = (i · (y1 z2 − z1 y2 )) − (j · (x1 z2 − z1 x2 )) + (k · (x1 y2 − y1 x2 ))

which simplifies to
 
y1 z2 − z1 y2
v × w = z1 x2 − x1 z2 
 
x1 y2 − y1 x2

This expression represents the cross product of v and w as a determinant.

• For any vectors u, v, and w in R3 , the following identity holds

(v · w)2 + (v × w)2 = ∥v∥2 ∥w∥2 (9.7)

• (Normal vector) Given three A, B, and C such that AB ∦ AC, the normal vector
to the plane containing these points can be found using the cross product

−−→ −→
n = AB × AC

This normal vector can be used in the equation of the plane ax + by + cz = d where
(a, b, c) are the components of n and d is found using one of the points and the
vector n.

• (Area of a Parallelogram) The magnitude of the cross product of two vectors v and
w in R3 gives the area of the parallelogram spanned by v and w

Area = ∥v × w∥ (9.8)

If AB and AC are two sides of a triangle, then the area of the triangle ABC is

1 −−→ −→
Area = ∥AB × AC∥ (9.9)
2
• (Volume of a Parallelepiped) The volume of a parallelepiped formed by three vectors
u, v, and w in R3 is given by the scalar triple product

Volume = |u · (v × w)| (9.10)


9.2 Subspaces and Spanning 83

• The volumn of a tetrahedron ABCD is given by

1 −−→ −→ −−→
Volume = |AB · (AC × AD)| (9.11)
6

9.2 Subspaces and Spanning


• A subspace of Rn is a subset U of Rn that satisfies the following properties:

1. U contains the zero vector 0.


2. If v and w are vectors in U , then v + w is also in U .
3. If v is a vector in U and c is a scalar, then cv is also in U .

• A set of vectors {v1 , v2 , . . . , vk } spans a subspace U of Rn if every vector in S can


be written as a linear combination of v1 , v2 , . . . , vk , that is, every vector v ∈ U can
be expressed as
v = α1 v1 + α2 v2 + · · · + αk vk ,
where α1 , α2 , . . . , αk are scalars.

9.3 Independence and Dimension


• A set of vectors {v1 , v2 , . . . , vk } in Rn is called linearly independent if the only
solution to the equation c1 v1 + c2 v2 + . . . + ck vk = 0 is c1 = c2 = . . . = ck = 0.

• A basis for a subspace U of Rn is a linearly independent set of vectors that span U .


That is, every vector in U can be expressed uniquely as a linear combination of the
vectors in the basis.

• The dimension of a subspace U of Rn , denoted as dim(U ), is the number of vectors


in any basis for U .
Example. Consider the vector space R3 . The subspace spanned by the vectors
v1 = [1 0 0]T and v2 = [0 1 1]T is a plane passing through the origin. It easy to
check that these vectors are linearly independent and span the plane. Therefore,
they form a basis for the subspace and the dimension of this subspace is 2. Consider
an m × n matrix A.

• The column space of A, denoted as col(A), is the subspace of Rm spanned by the


columns of A.

• The row space of A, denoted as row(A), is the subspace of Rn spanned by the rows
of A.

• The rank of a matrix A, denoted as rank(A), is the dimension of its column space or
equivalently the dimension of its row space. It is the maximum number of linearly
independent columns or rows in A.
1 0 1
 

Example. Consider the matrix A = −1 1 1.


 
0 1 2
84 THE VECTOR SPACE Rn

1 0 1
     

The column space of A is spanned by the vectors −1, 1, and 1.
     
0 1 2
h i h i h i
The row space of A is spanned by the vectors 1 0 1 , −1 1 1 , and 0 1 2 .
The rank of A is 2, as there are only two linearly independent rows (or columns).

9.4 Orthogonality

• (Orthogonality in Rn ) Two vectors u and v in Rn are orthogonal if their dot product


is zero
u⊥v⇔u·v=0

This means that the angle between u and v is π


2 radians (90 degrees), and they are
perpendicular to each other.

• (Orthogonal Sets and Orthogonal Basis) A set of vectors {v1 , v2 , . . . , vk } in Rn is


orthogonal if every pair of distinct vectors in the set is orthogonal.

• An orthogonal basis for a subspace U of Rn is a basis for U that is also an orthogonal


set.

9.5 Solved Problems

1°. (a) Find the plane ∆ passing through three points A(0, 1, 2), B(6, 3, 2), and C(1, −2, 0).
(b) Find the point of intersection of the plane ∆ and the line

2
     
x t
y  = 1 + −2t .
     
z 2 −t

(c) Find a vector equation of the line passing through M (2, 1, −1) and perpendicular
to ∆.
Solution. (a) To find the plane ∆ passing through three points A(0, 1, 2), B(6, 3, 2),
and C(1, −2, 0), we first compute

6−0 6
   
−−→ 
AB = 3 − 1 = 2 ,
  
2−2 0

1−0 1
   
−→ 
AC = −2 − 1 = −3 .
  
0−2 −2

The normal vector to the plane ∆, denoted by n, is given by the cross product of
−−→ −→
AB and AC, which is computed as (9.6)
9.5 Solved Problems 85

i 6 1
−−→ −→
n = AB × AC = j 2 −3 = −4i + 12j − 18k
k 0 −2

So, the normal vector to the plane ∆ is n = −4i + 12j − 18k = (−4, 12, −18)
Using vector form of the equation of a plane

n · (p − p0 ) = 0

where p is any point in the plane and p0 is any point on the plane. We can choose
any of the given points, let’s choose A(0, 1, 2).

−4(x − 0) + 12(y − 1) − 18(z − 2) = 0

2x − 6y + 9z = −12
So, the equation of the plane ∆ is 2x − 6y + 9z = −12.
(b) Let (x0 , y0 , z0 ) be the point of intersection of ∆ and the line.
Then, we have the following equations

x0 = 2 + t, y0 = 1 − 2t, z0 = 2 − t,
2x0 − 6y0 + 9z0 = −12.

Hence, we obtain
2(2 + t) − 6(1 − 2t) + 9(2 − t) = −12
and 14t = −28 or t = −2.
Thus, the point of intersection is (x0 , y0 , z0 ) = (0, 5, 4).
(c) Since the line is perpendicular to ∆, the normal vector of ∆ is also the direc-
tional vector of this line. Therefore, an equation of the line pasing through p0 and
perpendicular to ∆ is given by

p = p0 + tn.

where p0 is M .
A vector equation of the line is

2
     
x −4
y = 1 + t  12  ,
     
   
z −1 −8
where t is arbitrary real number.

2°. (a) Find the area of the triangle ABC, where A(0, 1, 2), B(1, 0, −1), C(2, 3, 2).
(b) Find the volumn of the terahedron ABCD, where A(0, 1, 1), B(2, −1, 2), C(0, 1, 3),
and D(3, 1, 4).
(c) Find the volumn of the parallelpiped formed by three vectors u = (2, −1, 1), v =
(1, 0, 3) and w = (0, 1, 1).
86 THE VECTOR SPACE Rn

Solution. (a) To fine the area of a triangle, we use the formula (9.9).
First, let’s compute

−−→ −→
AB = (1, −1, 3), AC = (2, 2, 0).

Next, we compute the cross product

−−→ −→
AB × AC = (−6, 6, 4).

The area of the triangle ABC is


1 −−→ −→ 1q √
∥AB × AC∥ = (−6)2 + 62 + 42 = 22.
2 2
(b) First, we calculate the following

−−→ −→ −−→
AB = (2, −2, 1), AC = (0, 0, 2), AD = (3, 0, 3).

The volume of the tetrahedron can be computed as (9.11)

1 −−→ −→ −−→ 1
V = |AB · (AC × AD)| = | − 18| = 3.
6 6
(c) The volume of the parallepiped is given by (9.10)

Volume = |u · (v × w)|

Since
2 1 0
 

u · (v × w) = det −1 0 1 = −4,


 
1 3 1
we obtain that the volume of the parallepiped is 4.
3°. (a) Find u · v if ∥u + v∥ = 7 and ∥u − v∥ = 5.
(b) Prove that ∥u + v∥2 + ∥u − v∥2 = 2∥u∥ + 2∥v∥2 for all vectors u, v ∈ Rn .
Solution. (a) We have the following

∥u + v∥2 = (u + v) · (u + v)
=u·u+u·v+v·u+v·v
= ∥u∥2 + 2u · v + ∥v∥2 (∗)
Similarly,
∥u + v∥2 = ∥u∥2 − 2u · v + ∥v∥2 (∗∗)

From (∗) and (∗∗) we have


∥u + v∥2 − ∥u − v∥2 = 4(u · v)

Hence, u · v = 14 (72 − 52 ) = 6.
(b) The conclusion is immediately drawn from part (a) if we subtract (∗∗) from (∗),
side by side.
9.5 Solved Problems 87

4°. (a) Show that U = {(x, y, x + y) | x, y ∈ R} is a subspace of R3 .


(b) Show that V = {(x, y, xy) | x, y ∈ R} is NOT a subspace of R3 .
Solution. (a) To show that U = {(x, y, x + y)} is a subspace of R3 , we need to
verify the three conditions for a subspace:
U contains the zero vector. The zero vector in R3 is (0, 0, 0), which belongs to
U since 0 + 0 = 0. Thus, U contains the zero vector.
U is closed under vector addition. Let (x1 , y1 , x1 + y1 ) and (x2 , y2 , x2 + y2 ) be
arbitrary vectors in U . Then, their sum is (x1 + x2 , y1 + y2 , (x1 + x2 ) + (y1 + y2 )).
Since (x1 + x2 ) + (y1 + y2 ) = (x1 + y1 ) + (x2 + y2 ), this sum is also in U . Therefore,
U is closed under vector addition.
U is closed under scalar multiplication. Let (x, y, x + y) be an arbitrary vector
in U and c be an arbitrary scalar. Then, c(x, y, x + y) = (cx, cy, cx + cy). Since
cx + cy = c(x + y), this scalar multiple is also in U . Therefore, U is closed under
scalar multiplication.
Since U satisfies all three conditions, it is indeed a subspace of R3 .
(b) V is not a subspace of R3 since we can show that V is not closed under the
vector addition. Indeed, let’s consider two specific vectors in V

u = (1, −1, −1), v = (2, 2, 4).

Then, their sum is u + v = (3, −1, −4) which is NOT in V since 3(−1) ̸= −4.

5°. (a) Find all value(s) of α such that the set {(1, 2, −1), (1, α, 2), (0, 1, 3)} is linearly
independent.
(b) Find m such that (m, 2, −1) belongs to the subspace spanned U = span((1, 0, −1), (2, 1, 1)).
Solution. (a) The determinant of the matrix formed by these vectors is given by

1 1 0
∆= 2 α 1
−1 2 3

= 3α − (6 + 1) = 3α − 7.

For the set to be linearly independent, the determinant is not equal to 0.


Therefore, we have
7
3α − 7 ̸= 0 =⇒ α ̸= .
3
So, all values of α except α = 7
3 make the set {(1, 2, −1), (1, α, 2), (0, 1, 3)} linearly
independent.
(b) (m, 2, −1) ∈ U if and only if (m, 2, −1) can be expressed as a linear combina-
tion of (1, 0, −1) and (2, 1, 1). That is, there exist a and b such that (m, 2, −1) =
a(1, 0, −1) + b(2, 1, 1) In other words, the system

a + 2b

 =m
0a + b = 2

−a + b = −1

88 THE VECTOR SPACE Rn

has at least one solution (a, b).


Solve this, we obtain m = 7.
6°. Find the dimension and basis of the solution space of the system

x − y + z + 2w =0
(

2x + y − z + w =0

Solution. Use Gaussian elimination to solve the system

" # " # " #


1 −1 1 2 0 1 −1 1 2 0 1 −1 1 2 0
−→ −→
2 1 −1 1 0 0 3 −3 −3 0 0 1 −1 −1 0
" #
1 0 0 1 0
−→
0 1 −1 −1 0

Hence, the general solution is

x = −t, y = s + t, z = s, w = t,
where t, s are arbitrary numbers.
Since every solution can be expressed as

[−t s+t s t]T = s[0 1 1 0]T + t[−1 1 0 1]T ,

the solution space is spanned by [0 1 1 0]T , [−1 1 0 1]T .


These two vectors are also independent. Therefore, {[0 1 1 0]T , [−1 1 0 1]T }
is a basis of the solution space.
Hence, the dimension of this space is 2.
7°. Find the dimension of the subspace U spanned by {(1, 0, −1, 2), (2, 3, 1, 1), (0, −5, −3, 3), (2, −2, −2, 4)}.
Solution. The subspace is spanned by 4 vectors in R4 . If these vectors are inde-
pendent, they form a basis for U .
Let’s check the independence of the vectors using elementary operations on the
matrix whose columns are these vectors.

1 2 0 2 1 2 0 2 1 2 0 2
     
 0 3 −5 −2   0 3 −5 −2   0 3 −5 −2 
 −→   −→ 
     
−1 1 −3 −2 0 3 −3 0 0 3 −3 0 
 
    
2 1 3 4 0 −3 3 0 0 0 0 0
1 2 0 2
 
 0 3 −5 −2 
−→ 
 
0 0 2 2 


0 0 0 0

The last matrix say that these vectors are not independent and we can choose exactly
3 independent vectors (three columns with leading ones) to form a basis for U .
Hence, dim(U ) = 3.
9.6 Supplementary Problems 89

8°. Find the dimension of the row and column spaces of the matrix

1 2 1 3
 

A = 2 1 −1 2  .
 
2 7 5 10

Solution. The dimension of the row space and column space of a matrix are equal
to the rank of the matrix.
We use elementary operations on rows to find the rank of the matrix.

1 2 1 3 1 2 1 3 1 2 1 3
     

2 1 −1 2 −→ 0 −3 −3 −4 −→ 0 −3 −3 −4
     
     
2 7 5 10 0 3 3 4 0 0 0 0

Hence, the rank of A is 2 and therefore, the dimension of the row and column spaces
of A is 2.

9.6 Supplementary Problems

1°. (a) Find the plane ∆ passing through three points A(0, 2, 3), B(−1, 0, 2), and C(7, 1, 0).
(b) Find the point of intersection of the plane ∆ and the line

4
     
x −t
y  =  1  +  2t  .
     
z −1 t

(c) Find a vector equation of the line passing through the origin and perpendicular
to ∆.

2°. (a) Find the area of the triangle OAB, where O(0, 0, 0), A(1, 0, 1), B(1, 0, 1).
(b) Find the volumn of the terahedron OABC, where O(0, 0, 0), A(0, 0, 1), B(0, 1, 1),
and C(1, 1, 1).
(c) Find the volumn of the parallelpiped formed by three vectors u = (1, −1, 1), v =
(1, 0, 1) and w = (0, 1, 2).

3°. (a) Suppose that {u, v, w} is an orthonormal set. Find (u − v) · (2u + w).
(b) Find all vectors orthogonal to both (1, 0, 1) and (2, −1, 2).

4°. Show that V = {(x, y, x2 ) | x, y ∈ R} is NOT a subspace of R3 .

5°. (a) Find all value(s) of α such that the set {(1, 0, 1), (α, 3, 3), (0, 1, −1)} is linearly
dependent.
(b) Find m such that (m, 2, −1) belongs to the subspace spanned U = span((1, 0, −1), (2, 1, 1)).

6°. Find the dimension and basis of the solution space of the system

x−y+z−w =0
(

2x − 2y − z + w =0
90 THE VECTOR SPACE Rn

7°. Find the dimension of the subspace spanned by {(1, 0, 2), (2, 1, 1), (3, 1, 3), (−1, 0, −2)}.

8°. Find the dimension of the row and column spaces of the matrix

1 −2 1
 
−2 3 −1
.
 
 0 2 3

0 1 4

9°. Suppose {v1 , v2 , v3 } is an independent set of vectors in Rn .


Set
v2 · b1
b1 = v1 , b2 = v2 − b1 ,
∥b1 ∥2
and
v3 · b1 v3 · b2
b3 = v3 − 2
b1 − b2 .
∥b1 ∥ ∥b2 ∥2

Show that {b1 , b2 , b3 } is an orthogonal set.

9.7 Multiple Choice Questions

1. Find an equation of the plane passing through three points O(0, 0, 0), P (2, 0, −1),
and Q(3, 3, 0).

(A) 2x + y + 4z = 0
(B) x − y + 2z = 0
(C) x − y + z = 0
(D) x + y + z = 0

2. Find the point of intersection of the plane x − 2y + z = 5 and the line



x

 =1+t
y = 3 − 2t

=2−t

z

(A) (1, −1, 2)


(B) (3, −1, 0)
(C) (0, −1, 3)
(D) (3, 1, 0)

3. Find the the cross product of u = (1, −2, 3) and v = (0, 1, −1).

(A) (1, −1, 1)


(B) (−1, 1, 1)
(C) (1, 1, −1)
(D) (−1, −1, 1)
9.7 Multiple Choice Questions 91

4. Find the area of the triangle ABC, where A(0, 1, 1), B(1, 2, 1), C(1, 3, −1).

(A) 5
(B) 3
(C) 5
2
(D) 3
2

5. Find the volumn of the pyramid ABCD, where A(0, 0, 0), B(1, 0, 0), C(1, 1, 1), and
D(1, 1, 3).

(A) 1
(B) 2
(C) 1
3
(D) 1
6

6. Find the volumn of the parallelpiped formed by three vectors u = (1, 1, 1), v =
(1, 0, 0) and w = (0, 1, 2).

(A) 1
(B) 2
(C) 1
3
(D) 1
6

7. Which of the following is NOT a subspace of R3 ?


U = {(x, y, xy)|x, y ∈ R},
V = {(x, y, x − y)|x, y ∈ R}.

(A) U only
(B) V only
(C) Both U and V
(D) Neither U nor V

8. Which of the following sets spans R3 ?


U = {(1, 0, 0), (0, 1, 0), (1, 1, 1)},
V = {(1, 1, 0), (0, 1, 1), (1, 2, 1)}.

(A) U only
(B) V only
(C) Both U and V
(D) Neither U nor V

9. Which of the following sets is linearly independent?

(A)
(B)
92 THE VECTOR SPACE Rn

(C)
(D)
   
 1
 −2 
10. What is the dimension of the subspace spanned by the set −2 ,  4  ?
   
 3

−6 

(A) 0
(B) 1
(C) 2
(D) 3

11. Find the dimension of the row and column spaces of the matrix

1 −2 1 0
 

2 −1 5 −1 .
 
5 −7 8 −1

(A) 4
(B) 1
(C) 3
(D) 2

12. Find all value(s) of α such that the set {(1, 2, −1), (0, 1, α), (−1, 1, 4)} is linearly
dependent.

(A) α = 2
(B) α = −2
(C) α = 1
(D) α = −1

13. Find m such that (1, m, −4) belongs to the subspace spanned U = span{(1, 3, −1), (2, 0, 1)}.

(A) 9
(B) 6
(C) 4
(D) 3

14. Find the dimension of the space spanned by {(1, 3, 0), (2, −1, 1), (3, 2, 1), (7, 7, 2)}.

(A) 4
(B) 1
(C) 2
(D) 3

ANSWER: 1B 2B 3B 4D 5C 6A 7B 8A 9C 10B 11D 12C 13A 14C


Bibliography
[1] Gilbert Strang and Edwin “Jed” Herman, Calculus Volumes 1, 2, Lyryx Learning
Inc, 2024.

[2] James Stewart, Calculus, Books/Cole, 2012.

[3] W. Keith Nicholson, Linear Algebra with Applications, Lyryx Learning Inc, 2021.

93
94 THE VECTOR SPACE Rn

You might also like

pFad - Phonifier reborn

Pfad - The Proxy pFad of © 2024 Garber Painting. All rights reserved.

Note: This service is not intended for secure transactions such as banking, social media, email, or purchasing. Use at your own risk. We assume no liability whatsoever for broken pages.


Alternative Proxies:

Alternative Proxy

pFad Proxy

pFad v3 Proxy

pFad v4 Proxy